You are on page 1of 99

µ somatic N.

si
ANTIFUNGAL/ ANTIVIRAL (POST TEST)
3. Mechanism of action of NNRTI
ANSWERS ALL LETTER A a. Bind directly to HIV 1 Reverse Transcriptase
1. Major extracellular site of action of antivirals resulting to RNA & DNA dependent DNA
a. Viral attachment and entry polymerase
b. Nucleic acid synthesis b. Inhibit post translational cleavage of gag –
c. Integration Transcription pol polyprotein resulting to immature non
d. Protein Synthesis infectious virus
e. Packaging and Assembly c. Competitive inhibition of HIV 1 reverse
transcriptase
DISCUSSION d. Binds to HIV 1 & 2 integrase enzyme
MAJOR SITES OF ACTION OF AV needed for the integration of the virus to
• Extra cellular the host
• Viral attachment & entry e. Binds to host CCR5 protein which facilitates
• Viral release entry of the virus inside the cell
• Intracellular
• Viral penetration DISCUSSION
• Uncoating MOA
• Nucleic acid synthesis PI - Inhibit post translational cleavage of
• Integration transcription gag – pol polyprotein resulting to
• protein synthesis immature non infectious virus
• Packaging & assembly NNRTI - Bind directly to HIV 1 Reverse
Transcriptase resulting to RNA & DNA
2. Mechanism of action of Protease Inhibitors dependent DNA polymerase
a. Inhibit post translational cleavage of NRTI - Competitive inhibition of HIV 1
gag – pol polyprotein resulting to reverse transcriptase
immature non infectious virus InSTI - Binds to HIV 1 & 2 integrase enzyme
b. Bind directly to HIV 1 Reverse needed for the integration of the virus to
Transcriptase resulting to RNA & DNA the host
dependent DNA polymerase Entry Inhibitors - Binds to host CCR5
c. Competitive inhibition of HIV 1 reverse protein which facilitates entry of the virus
transcriptase inside the cell
d. Binds to HIV 1 & 2 integrase enzyme
needed for the integration of the
virus to the host 4. Entry inhibitor
e. Binds to host CCR5 protein which a. Maraviroc
facilitates entry of the virus inside the b. Ritonavir pi
cell c. Efavirenz NNRTI
d. Zidovudine NRTI
DISCUSSION e. Abacavir NRTI
MOA
PI - Inhibit post translational cleavage DISCUSSION
of gag – pol polyprotein resulting to Maraviroc – Entry inhibitor
immature non infectious virus Ritonavir – PI
NNRTI - Bind directly to HIV 1 Reverse Efavirenz – NNRTI
Transcriptase resulting to RNA & DNA Zidovudine – NRTI
dependent DNA polymerase Abacavir – NRTI
NRTI - Competitive inhibition of HIV 1
reverse transcriptase
InSTI - Binds to HIV 1 & 2 integrase 5. Used in the treatment of Hepatitis B infection
enzyme needed for the integration of a. Entecavir
the virus to the host b. Daclatasvir
Entry Inhibitors - Binds to host CCR5 c. Ledipasvir
protein which facilitates entry of the d. Ombitasvir
virus inside the cell e. Dasabuvir
DISCUSSION e. Isavuconazole
Anti HBV Drugs
• Adefovir dipivoxil 10. Mechanism of action is blockade of B Glucan
• Entecavir Synthesis
• Lamivudine a. Amphotericin B
• Telbivudine b. Flucytosine
• Tenefovir disoproxil c. Ketoconazole
ALL the other choices are Anti HCV d. Nystatin
e. Terbinafine
6. Primarily used for CMV retinitis
a. Ganciclovir CASE
b. Acyclovir 11. Clinical presentation of case 1 with Oropharyngeal
c. Valacyclovir Candidiasis
d. Famcyclovir a. White patches covering the buccal mucosa,
e. Pencyclovir tongue and posterior pharyngeal wall.
b. Yellowish mucopurulent discharge in the
DISCUSSION post pharyngeal wall
c. Bleeding ulcerated lesions on the tongue
CMV Retinitis Drugs and oropharynx
• Ganciclovir d. Erythematous buccal mucosa and
• Valganciclovir oropharyngeal wall
• Foscarnet e. Elevated hyperemic patches on the tongue,
• Cidofovir buccal mucosa and oropharynx
The rest are for HSV & VZV
• Acyclovir 12. Topical drug of choice for Oropharyngeal Candidiasis
• Valacyclovir a. Nystatin
• Famciclovir b. Amphotericin B
• Penciclovir c. Ketoconazole
• Docosanol & Trifluridine d. Griseofulvin
e. Micafungin
7. Neuraminidase inhibitor used for both
Influenza A & B 13. MOA of the topical drug of choice for
a. Peramivir Oropharyngeal Candidiasis
b. Ribavirin a. Forms pores in cell wall membrane
c. Elbasvir b. Blocks fungal CYP 450
d. Dasabuvir c. Blocks B Glucan synthase
e. Ledipasvir d. Inhibit squalene epoxidation
e. Interfere with fungal DNA and RNA
DISCUSSION synthesis
Anti influenza Drugs
• Oseltamivir & Zanamivir 14. Alternative topical drug for Oropharyngeal
• Peramivir Candidiasis
• Amantadine & Rimantadine a. Clotrimazole
b. Amphotericin B
8. Azole of choice for Aspergillus infection c. Griseofulvin
a. Voriconazole d. Caspofungin
b. Itraconazole e. Micafungin
c. Fluconazole
d. Posaconazole 15. Diagnosis for the case with Erythematous
e. Isavuconazole maculopapular rashes on the thoracic with
occasional fluid filled blisters accompanied by
9. Broadest spectrum Azole and the first azole moderate to severe pain
with significant activity vs. Mucormycosis a. Shingles
a. Posaconazole b. Measles
b. Voriconazole c. HIV
c. Itraconazole d. CMV
d. Fluconazole e. Hepatitis C
16. True statement regarding the case with
Erythematous maculopapular rashes on the
thoracic area with occasional fluid filled blisters
accompanied by moderate to severe pain
a. Antiviral creams are generally not
recommended
b. Oral Abacavir is the drug of choice
c. Oral Antibiotics are given as
prophylaxis to prevent secondary
bacterial infection
d. Trifluridine creams are applied to the
lesions within 12 hours to shorten
healing time
e. Cytomegalovirus is the causative agent

17. Used in the treatment of Herpes Zoster or


Varicella Zoster infection
a. Acyclovir
b. Abacavir
c. Tenofovir
d. Atazanavir
e. Indinavir

18. Causative organism in the case with edematous


external auditory canal and fuzzy black cotton
like material
a. Aspergillus
b. Candida
c. Clostridium
d. Cryptococcus
e. Blastomyces

19. Drug of choice for Otomycosis (case 2)


a. Clotrimazole
b. Amphotericin B
c. Ketoconazole
d. Griseofulvin
e. Terbinafine

20. MOA of the drug of choice for Otomycosis


(Case 2)
a. Reduce ergosterol synthesis by
inhibiting fungal CYP 450
b. Forms pores in fungal cell wall
c. Blocks B Glucan synthase
d. Inhibit squalene epoxidation
e. Interfere with fungal DNA and RNA
synthesis
MD2024 | PLATINGS WITH RATIO

PHARMACOLOGY
USE AT YOUR OWN RISK, BESTIE

NSAIDS POST-TEST 3. NSAID bioavailability is not changed by food intake:


1. The following are non-selective COX inhibitors, a. Naproxen
except b. Fenoprofen
a. Piroxicam - non c. Oxaprozin
b. Ibuprofen d. Paracetamol
c. Mefenamic Acid
d. Meloxicam NSAIDs are weak organic acids, except for
Nabumetone. They are rapidly absorbed in GIT, but
Meloxicam is a selective COX-2 inhibitor. This includes there are two drugs whose bioavailability are not
celecoxib, valdecoxib, rofecoxib, parecoxib, etc. affected by food: Fenoprofen and Piroxicam
REMEMBER: -coxib = selective COX-2 inhibitors
4. The ff are TNF-alpha, except:
Piroxicam, Ibuprofen, and Mefenamic are non- a. Infliximab
selective. They both act on COX-1 and COX-2 b. Tocilizumab
c. Etanercept
Acetaminophens and Paracetamol are not NSAIDS. d. Adalimumab
They are para-amino phenols.
These drugs are known as DMARDs (Disease Modifying
Aspirin, Sodium Salicylate, Salsalate, Diflunisal are Anti-Rheumatic Drugs)
salicylic acid derivatives, so they inhibit irreversibly
prostaglandin synthesis Conventional DMARDs:
Methothrexate Azathioprine
Indomethacin and Sulindac are para-chlorobenzoic Cyclophosphamide Cyclosporine
acid derivatives or indoles Leflunomide Sulfasalazine
Mycophenolate mofetil
Phenylbutazone and Azapropazone are Pyrazalone Chloroquine & Hydrochloroquine (anti-malarial
derivatives. effect)

2. The ff are major effects of NSAIDS, except: Biologic DMARDs:


a. Anti-inflammatory TNF-alpha Infliximab Etanercept
b. Analgesic Neutralizing Adalimumab Certolizumab
c. Tocolytic Agents Golimumab
d. Antipyretic Anakinra
IL-1 Neutralizing
Canakinumab
Agents
NSAIDs have 4 major effects: Rilonacept
Anti-inflammatory Depletes B-cells Rituximab
Antipyretic Interferes w/ T cell Abatacept
Analgesic activation
Anti-platelet Anti-IL-6 Receptor Tocilizumab
Due to inhibition of cyclooxygenase pathway, inhibiting Antibody
prostaglandin and thromboxane synthesis

Tocolysis: There are some drugs with tocolytic activity, 5. Which of the ff is correct regarding DMARDs
observe this in a. Might arrest or slow the progression of bone in
NSAIDs with indole derivatives, such as Sulindac and cartilage destruction
Indomethacin b. Effects may take 6 weeks to 6 mos to become evident
c. Exert minimal direct non-specific anti-inflammatory
analgesic effect
d. All the above

1
6. This drug is used in gouty arthritis to relieve pain by 10. MT is a 10-y/o boy who is bought into the ED by his
inhibiting leukocyte migration and phagocytosis: parents, diagnosed with a bronchial asthma attach.
a. Colchicine
aggressive conventional therapy with short acting
The action of Colchicine is to reduce or inhibit the beta 2 agonist and systemic corticosteroids. He is
migration; its effect is on the microtubules. It will admitted to the hospital and placed in the ICU. It is
relieve the pain by binding to your tubulin dimers, decided to try a trial dose of Theophylline, which has a
preventing the polymerization of microtubules. Walang narrow therapeutic window of 5-15 ug/ml. what toxic
microtubule, walang locomotion
theophylline levels become too high?
7. The newest urate-lowering therapy for refractory
gout which is contraindicated in patient with: a. Sedation
a. Gastritis b. Seizures and arrythmias
b. Heart failure c. Bradycardia
c. G6PD deficiency d. Hyperkalemia
d. Porphyria
Sa CVS, ang pinaka common is arrythmia.
Uricosuric agents are contraindicated in patients with Sa CNS, seizure.
G6PD deficiency.
CLINICAL CASE: NSAID POST TEST
8. Which is most potent and effective controller 1. Drugs of choice for producing a rapid relief from
medication of asthma? bronchoconstriction:
a. Intake of anti-muscarinic drugs a. Inhaled corticosteroids
b. Inhaled beta 2 agonists b. Inhaled anti-muscarinic agents
c. Inhaled corticosteroids c. Inhaled cromolyn sodium
d. All are correct d. Inhaled short acting beta 2 agonist

Dalawa kasi yun, controller and reliever. Bronchodilators = Inhaled short acting beta agonist
Controller = Anti-inflammatory corticosteroids
Reliever = Bronchodilators 2. Possible trigger in the case of FDM:
a. Respiratory infections
Since the question is controller, the answer must be an b. Specific allergen
anti-inflammatory agent, such as inhalation c. Extreme temperature
corticosteroids: d. Strong emotions
Beclometasone Budesonide Ciclesonide
Flunisolide Mometasone Triamcinolone Respiratory infection because patient had history of
bronchitis.
If reliever, release inhibitors yan:
Cromolyn Sodium Nedocromil
a. Moderate to severe respiratory distress with bilateral
or Leukotriene Antagonists: expiratory wheezing
Zeliuton Montelukast Zafirlukast b. Mild respiratory distress with bilateral expiratory
wheezes
Anti-inflammatory agents that will relieve inflammation c. Moderate to severe respiratory distress with bilateral
= CONTROLLER inspiratory wheezing
d. Severe respiratory distress with bilateral inspiratory
Anti-inflammatory agents that will provide rapid relief wheezes
= BRONCHODILATOR (short-acting drugs)
4. Antidote for acetaminophen/paracetamol overdose:
9. Short acting beta 2 agonists are administered via: a. All are correct
a. All are correct b. N-acetyl cysteine
b. Metered dose inhaler c. Mucomyst
c. Nebulizers d. Oral methionine
d. Oral administration
Oral methionine is just as effective as N-acetylcysteine
as an antidote. Mucomyst is also an N-acetyl cysteine

2
5. Minor but highly active metabolites/s of AUTACOIDS PLATING
acetaminophen/paracetamol responsible for liver and 1. The following is/are used to treat pulmonary
kidney toxicity: hypertension and Porto pulmonary hypertension
a. Glucuronide a. Iloprost
b. Acetyl-p-benzoquinone b. Epoprostenol
c. Glucuronide and acetaminophen SO4 c. All are correct
d. Acetaminophen SO4 d. Treprostinil

6. What contributes to further depletion of hepatic 2. The eicosanoid that is a potent vasoconstrictor and
glutathione reserve in patient FDM? a smooth muscle cell mitogen
a. Intake of other NSAIDS a. PGE2
b. Infection b. Thromboxane A2
c. Alcohol c. PGI2
d. Diabetes d. PGF2 alpha

Alcohol depletes glutathione. 3. 5 lipoxygenase inhibitor/s


a. All are correct
If chronic, alcohol induces the activity of P450 b. Zafirlukast
c. Montelukast
7. What is the complaint of FDM with regard to her acute d. Zileuton
gouty arthritis?
a. Pain on the big toe, right 4. This drug can be used for the treatment of eyelash
b. Acute pain in her right elbow hypotrichosis and alopecia
c. Back pain a. Iloprost
d. Acute intense pain b. Bimatoprost
c. Travoprost
8. A purine oxidase inhibitor taken by FM for her gouty d. Latanoprost
flare prevention.
a. Febuxostat 5. Powerful bronchodilator
b. Allopurinol a. PGF2
b. Thromboxane A2
Febuxosat is also a xanthine inhibitor, but the question c. All are correct
is the one taken by the patient, which is Allopurinol d. PGE2

9. Considered the first line of therapy for acute gouty 6. Aspirin and other NSAIDS
flares: a. Increased thromboxane A2
a. Colchicine b. Increased leukotriene
b. NSAIDS c. All are correct
c. Methotrexate d. increased prostaglandin synthesis
d. Aspirin low dose
Not prostaglandin and thromboxane because they are
Kapag acute gout flare, the first line of therapy is inhibited. The answer is increased leukotriene. Kapag
NSAID because you have to relieve the pain and iniinhibit mo yung cyclooxygenase, ang na-
inflammation. lipoxygenase pathway. So dadami ang leukotrienes.

Aspirin, no. You can also give Colchicine, but it is not 7. An INCORRECT statement about NITRIC OXIDE:
the first line, but you still give it together with NSAID a. It functions as an anesthetic
b. It functions as a vasodilator
10. This is an effective treatment to prevent recurrent c. It is a neurotransmitter and an immune regulator
gouty flares, but is not for use during acute flares because d. It inhibits platelets
as monotherapy it makes the symptoms worse?
a. Allopurinol Nitric Oxide is a potent vasodilator
Nitric Oxide is considered a neurotransmitter
Nitric Oxide inhibits platelets
Nitrous Oxide is the one who functions as an anasthetic

3
8. The following are major targets of Nitric Oxide, 14. It is first nonpeptide renin inhibitor approved for
except: the treatment of hypertension
a. Metalloproteins a. Aliskerin
b. Thiols b. ACE inhibitor
c. Tyrosine nitration c. Beta blocker
d. Xanthine Oxidase d. ARB

9. Nitric oxide role in disease, EXCEPT: ACE = ace inhibitor angiotensin converting enzyme
a. Release of iNOS by cytokines in septic shock ARB = receptor inhibitor
b. Vasoconstriction in vascular smooth muscle
c. Excessive NO synthesis in neurologic disease 15. It is a second generation B2 receptor antagonist
d. Activation of COX-2 during inflammation effective in the treatment of hereditary angioedema:
a. Esmolol
Nitric oxide is a potent vasodilator b. Nadolol
c. Propranolol
10. This antihypertensive drug blocks B1 receptor and d. Icatibant
produces vasodilation thru release of Nitric oxide:
a. Propranolol 16. Which of the following is true regarding
b. Sildenafil vasopressin?
c. Nebivolol a. Promote water excretion
d. Carvedilol b. G-protein coupled receptor
c. Vasodilator
Propanolol is not a B1 receptor. Non-selective. d. V1A mediates vasodilation
Sildenafil is a phosphodiesterase inhibitor.
Nebivolol is a beta blocker with vasodilating effect Vasopressin inhibits water excretion
Carvedilol is non-selective w/ Alpha 1 blocking activity Vasoconscrictor
V1B mediates vasodilation. V2 has antidiretic effect.
11. This nitric oxide donor is used for rapid pressure
reduction in arterial hypertension It is a g-protein coupled receptor because it is linked to
a. Amyl nitrite adenylate cyclase
b. Sodium nitroprusside
c. Nitroglycerin 17. True of Histamine:
d. Sildenafil a. Histamine plays an important role in gastric acid
secretion
12. It stimulates Renin K indirectly by alpha adrenergic b. The main function as a neurotransmitter and a
activation of the renal baroreceptor in macula densa. neuromodulator
a. Renal baroreceptor c. Histamine occurs in plants, animal, venoms stinging
b. Angiotensin secretions
c. Sympathetic Nervous System d. All of the above
d. Intracellular signaling pathway
18. Drugs that can displace and release histamine from
13. Angiotensinogen is synthesized in this organ: its bound form within cells
a. Pancreas a. Paracetamol
b. Kidneys b. Compound 48 k-60
c. Liver c. Mefenamic acid
d. Lungs d. Morphine

Lungs = ACE 19. Upon injury to a tissue, released histamine caused


Kidneys = Renin a. Release of lysosome contents
Liver = Angiotensinogen b. All are correct
Pancreas = Insulin, Glucagon, Pancreatic Polypeptide c. Local vasoconstriction
d. Leakage of plasma containing mediators of acute
inflammation

4
3. This drug can be used as a diagnostic test for
20. Mechanism of action of histamine myasthenia gravis
a. There are five different histamine receptors that have a. Edrophonium
been characterized b. Ecothiopate
b. Histamine combines with specific receptors located in c. Neostigmine
the nucleus d. Pyridostigmine
c. Some antihistamines previously considered to be
traditional pharmacologic antagonist are now considered Tensilon test or Edrophonium test is used to evaluate
to be inverse agonists MG. It involves an injection of Tensilon (edrophonium),
d. All are correct. after which your muscle strength is evaluated to
determine whether your weakness is caused by
Histamine combines with specific receptors in the myasthenia gravis or not.
membrane, not nucleus.
There are only 4 Histamine receptors 4. This drug can be used for anticholinergic poisoning
a. Physostigmine
21. H2 receptor distribution b. Neostigmine
a. Gastric mucosa c. Pilocarpine
b. Cardiac Muscle d. Edrophonium
c. Brain
d. All are correct 5. A synthetic drug with partial agonist action at K
alpha-4 beta-2 nicotinic receptors that can be used in
22. Inverse agonist H4 receptor: smoking cessation
a. Cetirizine a. Varenicline
b. Thioperamide b. Tri-O-cresyl phosphate
c. Clozapine c. Bethanechol
d. Cimetidine d. Pilocarpine

23. Side effects of histamine: 6. A muscarinic agonist that activates M1, M2, M3 in all
a. All are correct peripheral tissues causing increased secretion,
b. Dilation of intestinal smooth muscle smooth muscle contraction, except the vascular
c. Inhibition of gastric acid secretion smooth muscle and changes in heart rate. This is used
d. Constriction of bronchiolar smooth muscle in post operative in your neural neurogenic ileus and
urinary retention
24. Intradermal injection of histamine causes a. Varenicline
a. Red spot b. Tri-O-cresyl phosphate
b. All are correct c. Bethanechol
c. Edema d. Pilocarpine
d. Flare response
7. Which of the following is not an adreno receptor:
ANS DRUGS PLATINGS a. Alpha adrenoreceptor
1. This drug is sometimes considered as irreversible b. Beta
cholinesterase inhibitor due to its long duration of c. Dopamine
action d. Nicotinic
a. Echothiopate
b. Neostigmine Nicotinic is cholinergic.
c. Pyridostigmine
d. Edrophonium 8. Ligand-biding to alpha 1 lead to:
a. Inhibition adenylate cyclase
2. This is the most widely used cholinesterase inhibitor b. Stimulation of adenylyl cyclase
for the management of paralytic ileus or urinary c. Formation of IP3 and DAG
retention d. Opening of sodium potassium channels
a. Neostigmine
b. Cevimeline Formation of IP3 and DAG, which is why it promotes
c. Pilocarpine vasoconstriction
d. Methacholine

5
9. Stimulation of beta 1 receptor in the kidney lead to: 15. The following are the usual effects of
a. Increased sodium excretion parasympatholytic drugs
b. Increased renin secretion a. Bronchoconstriction
c. Activation of aldosterone secretion b. Bronchial secretion
d. Hypotension c. Decrease laryngospasm
d. All of the above
Sa kidney, tatlo lang yun.
- Renin release Bronchoconstriction, iblock mo bronchodilation
- Sodium and water reabsorption, hindi excretion Bronchial secretion, iblock mo decrease bronchial
secretion
10. Receptors for the following neurotransmitters are Decrease laryngospasm kasi nagbbronchodilate yung
found in the adrenergic neurons, except: area
a. Acetylcholine
b. Norepinephrine 16. The following are adverse effects of atropine
c. Dopamine a. Dry mouth
d. Serotonin b. Hallucination
c. Hot flushes
The answer is serotonin kasi sa sympathetic d. All of the above
neurotransmitter adrenergic, meron ka rin dun
cholinergic, lalo yung post-ganglionic. Lahat kasi yun 17. The ff are therapeutic effects of anti-muscarinic
cholinergic. drugs

11. Neurotransmitter that has excitatory effect when b. COPD


it binds to beta 2: c. Motion sickness
a. Acetylcholine d. All of the above
b. Epinephrine
c. Histamine 18. The ANS is:
d. Norepinephrine a. Its activities are not under direct conscious control
b. Concerned primarily with integration of visceral
Effect of epinephrine is primarily on beta 2 functions
c. Involved with somatic functions
12. The following drugs are anti muscarinic d. A and B
a. Atropine
b. Scopolamine 19. The general class of adrenoreceptor include the ff:
c. Hyoscine a. Alpha
d. All of the above b. Beta
c. Dopamine
13. The following are the usual of ophthalmologic d. All of the above
effects of antimuscarinic
a. Mydriasis 20. The primary transmitter molecule released from
b. Cycloplegia parasympathetic postganglionic:
c. Decrease lacrimal secretion a. Acetylcholine
d. All of the above b. Norepinephrine
c. Dopamine
Mydriasis is correct kasi anti-muscarinic. d. Muscarine
Muscarinic = Constriction
Anti = Dilatation Kapag parasympathetic pre-ganglionic = Acetylcholine
Parasympathetic post-ganglionic = Acetylcholine din
Cycloplegia or loss of accommodation is correct.
Decrease lacrimal secretion is correct. Norepinephrine ay post-ganglionic ng sympathetic

14. The following are the CVS usual effects of Muscarine is a substance that acts on muscarinic
cholinergic blocking drugs receptors
a. Tachycardia
b. Coronary vasodilation
c. Skeletal muscle vasodilation
d. All of the above

6
21. The enzyme that catalyzes the synthesis of 4. During prolonged surgical procedure, patient was
Acetylcholine in the cytoplasm is: noticed to have a fall in blood pressure. The
a. Choline Acetyltransferase anesthesiologist administered a pressure agent by
b. Acetylcholinesterase intravenous bolus. What is this drug?
a. Doxazosin
Acetylcholinesterase DESTROYS the synthesis b. Phentolamine
c. Isoproterenol/Isoprenaline
22. The vesicular monoamine transporter can be d. Phenylephrine
inhibited by this drug, thereby inhibiting uptake of
biogenic amines 5. CD, 20 y/o patient, brought in the ER because of
a. Reserpine severe urticaria and severe hypotension. Which of the
b. Vesamicol following is a drug of choice for treating its life-
c. Bretylium threatening condition?
d. Cocaine a. Epinephrine
b. Terbutaline
Cocaine and bretylium sa uptake. c. Isoproterenol
Vesamicol sa entry into the cell, hindi sa vesicle. d. Phenylephrine

23. The enteric nervous system includes: 6. A 30-year-old male was brought in because of
a. Auerbach cardiogenic shock. He was given beta 1 selective
b. agonist to increase cardiac output in contractility. This
c. Celiac plexus
d. A and B only a. Epinephrine
b. Norepinephrine
c. Dopamine
d. Dobutamine
SYMPATHETIC NERVOUS SYSTEM PLATINGS
Epinephrine and Norepinephrine are not selective to
1. Activation of alpha receptor leads to which of the ff:
beta 1. Dopamine is not selective to beta 1.
a. Arterial and Venous Vasoconstriction
Dobutamine is beta 1 receptor selective agonist
b. Arterial Vasoconstriction and Venous dilatation
c. Arterial Vasodilation and Venous dilatation
7. Clinical use a beta blocker:
a. Hypertension
The most important effect of alpha 1 is
b. Prostatitis
vasoconstriction. Dapat parehas may vasoconstriction.
c. Erectile dysfunction
d. Hypotension
2. AB, 28 y/o woman, with history of hypertension,
e. Prostatic hyperplasia
who is planning to become pregnant in the near future,
comes into your office to discuss the therapeutic
8. Most common adverse effect of beta blocker in the
options to replace ACE inhibitor with a drug relatively
heart
safe in pregnancy. What do you think is the drug?
a. Bradycardia
a. Prazosin
b. Tachycardia
b. Methyl DOPA
c. Angina
c. Guanabenz
d. Cardiomegaly
d. Clonidine
e. Arrythmia
3. The following are direct acting sympathomimetic
Bradycardia because you decrease the activity of the
drugs, except:
heart.
a. Midodrine
b. Oxymetazoline
Wrong ang tachcardia. Wrong ang angina because it
c. Tyramine
can also be treated with beta blocker.

7
9. Which of the following statement is 14. The following are adverse effect of alpha 1
a. Labetalol is both alpha and beta blocker antagonist effect, except:
b. Carvedilol is a selective beta blocker used extensively in a. Myosis
heart failure b. Reflex tachycardia
c. Propranolol is best given to control hypertension in c. Urinary retention
asthmatic d. Orthostatic hypotension
d. Phenoxybenzamine reversibly blocks both alpha and
beta Myosis, yes, kasi ibblock yung radial muscle.
e. Yohimbine is given to decrease heart rate and blood Orthostatic hypotension, yes, ito yung nagccause ng
pressure reflex tachycardia.

Labetalol is one of the beta blockers with vasodilating Not urinary retention kasi treatment ang alpha 1
effect, together with Nebivolol. antagonist sa urinary retention.
Hindi pwede ang propranolol because non-selective 15. A patient was brought to the emergency room due
ito, ibblock B2, mag-bronchoconstrict. to difficulty of breathing after intake a beta blocker
for hypertension. Which of the following beta blocking
Phenoxybenzamine yung ginagamit sa agent most likely caused the condition?
pheochromocytoma kasi irreversible ang blockage. a. Propranolol
b. Atenolol
10. Oral drug given for patients with open angle c. Nebivolol
glaucoma c. Metoprolol
a. Acetazolamide
b. Pilocarpine Nag-bronchoconstrict, so dapat non-selective, which is
c. Apraclonidine propranolol.
d. Latanoprost
16. Beta blocker with selective alpha 1 antagonist
Acetazolamide is one of the most important indication effect:
of carbonic anhydrase inhibitor a. Labetalol
b. Carvedilol
11. Selective beta 1 blocker c. A and B
a. Metoprolol
b. Propranolol 17. Which of the ff alpha-adrenergic antagonist is used
c. Timolol in the treatment of orthostatic hypotension?
d. Carvedilol a. Urapidil
b. Indoramin
Metoprolol is a beta 1 selective blocker c. Yohimbine
d. Prazosin
12. Non-selective beta blocker
a. Propranolol 18. Which of the ff is a mixed-antagonist
b. Atenolol adrenoreceptor?
c. Esmolol a. Labetalol
d. Nebivolol
e. Metoprolol 19. Stimulation of this adrenoreceptor leads to
formation of IP3 and DAG to increase intracellular
The other choices are selective beta 1 blockers calcium?
a. Alpha 1
13. Alpha 1 antagonist are least likely used in the b. Beta 1
treatment of what? c. D2
a. Hypertension
b. Raynaud's phenomenon Alpha 1. That is the reason why it causes
c. Benign Prostatic Hyperplasia (BPH) vasoconstriction.
d. Tachycardia

Beta blocker ang tachycardia

8
20. Stimulation of this adrenoreceptor leads to 4. Steps in basic drug evaluation that is considered the
inhibition of adenylyl cyclase? most expensive because it requires a thorough
a. Alpha 2 evaluation through the use of animal studies to
determine acute dose, chronic dose, etc:
21. Stimulation of this adrenoreceptor leads to a. 1
stimulation of adenylyl cyclase and increased CAMP b. 2
a. Beta 1 c. 3
d. 4
Kapag alpha 1, IP3 and DAG
Kapag beta 1 and 3, increased activity of adenylyl 5. Which of the following is NOT a true statement?
cyclase a. Drugs with molecular weight of <10,000 is more
selective in binding with a receptor and specific in its
22. This drug is an example of mixed alpha and beta action.
agonist b. Hydrophobic bonds are the weakest because drugs
a. Norepinephrine bounded
b. Clonidine by this type would require a specific for to a receptor
c. Dobutamine c. To accelerate the clearance of a basic drug, it is
d. Fenoldopam necessary to adjust the pH of the urine to alkaline
Fenoldopam is dopamine agonist. d. Most drugs exist in race mixtures
Dobutamine is selective beta 1.
6. TRUE statement, EXCEPT:
23. Stimulation of this adrenoreceptor will lead to a. A receptor has a nonfunctional form (R1) and an active
increased CAMP site (R2)
a. D1 b. A drug may interact with a receptor through the active
b. D2 site, allosteric inhibitor and allosteric activation sites
c. D3 c. Receptors are component of a cell that interact with a
d. D4 drug molecule to produce an effect
d. Drugs with MW 20,000 to 30,000 permeates a
24. This is an example of dopamine agonist membrane through facilitated diffusion
a. Fenoldopam
7. True statement of an inducer, EXCEPT:
GENERAL PRINCIPLES PLATING a. An inducer accelerates substrate metabolism
1. Drug with a therapeutic index of near the value of >2 b. Smoking, eating char-broiled foods and other
is: environmental pollutants are examples of an inducer.
a. Unsafe for use in humans c. An inducer decreases pharmacologic action of the drug
you combine it with.
b. Has a narrow margin of safety
d. Cimetidine enhances the metabolism of Atorvastatin.
c. Given more frequently
d. Has a wide margin of safety
8. True statement, EXCEPT:
a. Delayed effect of a drug may be due to a slow turnover
2. Ototoxicity after prolonged use of aminoglycosides
of physiologic substance that is involved in the expression
or hepatotoxicity after chronic use of statins are
of the drug effect
examples of this adverse drug reaction:
b. The desirable to administer a loading dose to promptly
a. Type A
b. Type B raise the drug concentration in plasma to target
c. Type C
d. Type D c. Clearance is the single most important factor in
determining drug concentration
d. A low albumin concentration may result in lower drug
3. An example of a phase I of drug metabolism is where
concentration for drugs such as phenytoin and salicylates
a functional group is unmasked, EXCEPT:
a. Deamination
b. Oxidation
c. Acetylation
d. Microsomal oxidases

9
9. The following are characteristics of a receptor, 16. INCORRECT statement about volume of
EXCEPT: distribution:
a. Receptors largely determine the quantitative a. It is the amount of apparent space in the body that is
relationship between dose or concentration of a drug and able to contain a drug
pharmacologic effects. b. The higher the Vd, more drug is in the extravascular
b. Receptors are responsible for selectivity of drug action. tissues
c. Receptors mediate the action of pharmacologic c. It relates to the amount of drug in the body to the
agonists and antagonists. concentration of the drug in blood or plasma
d. Structural proteins are the best-characterized drug d. It is the fraction on unchanged drug reaching the
receptors. systemic circulation bioavailability

10. CYPD2D6 is responsible for the O-demethylation 17. This signaling mechanism is utilized by beta
of codeine to morphine and persons with variations on adrenergic agonists and adenylyl cyclase:
this phenotype (whether PM, IM or UM) would result to a. Intracellular receptors
different responses towards codeine. this variation to b. Voltage gated
drug response is due to: c. G-protein
a. Genetic variations in enzymes d. Ligand gated
b. Genetic variations in transporters
c. Genetic variations in immune system functions 18. A type of drug permeation for substances that are
d. Polygenic effects so large or impermeable that they can only enter when
a cell engulfs the drug and carries it into the cell by
11. This refers to the maximal response that can be pinching off the newly off of newly formed vesicle into
produced by a drug: the membrane:
a. E a. Simple diffusion
b. Emax b. Active transport
c. EC50 c. Endocytosis
d. Vmax d. Exocytosis

12. The following are measured by quantal dose-effect 19. An INCORRECT statement about
curve, EXCEPT: pharmacogenomics:
a. EC50 a. It is the study of genetic factors that underlie variations
b. Emax of drug response
c. TI b
d. TD50 c to
guide drug and dosing selection
13. Quantitative variations in drug response includes d. It studies the genetic sequence of bacteria and
the following, EXCEPT: viruses in the development of vaccines
a. Hypersensitivity
b. Hyporeactive 20. The following are factors that affect drug
c. Tolerance absorption, EXCEPT:
d. Tachyphylaxis a. Drug solubility
b. Liver disease affects drug distribution and metabolism
14. Which DO NOT exhibit capacity-limited c. Surface area
elimination: d. Blood flow
a. Lidocaine
b. Phenytoin 21. A branch of pharmacology that study the cost
c. Ethanol effectiveness of a particular therapeutic regimen:
d. Aspirin a. Pharmacodynamics
b. Pharmacogenomics
15. Drug A reduces BP at a dose of 5mg while Drug B c. Pharmacotherapeutics
reduces BP at a dose of 10mg. This statement shows d. Pharmacoeconomics
that:
a. Drug A is more potent than drug B
b. Drug B is more efficacious than drug A.
c. Drug A has a narrow margin of safety than Drug B
d. Drug B is absorbed faster than Drug A

10
22. A drug that interacts with a receptor, activate the
receptor but produce a lower response at full receptor
occupancy due to low intrinsic efficacy:
a. Full agonist
b. Partial agonist
c. Competitive antagonist
d. Physiologic antagonist

23. A drug interaction seen when 2 drugs are taken


together and the effect that they produce is opposite
of each other:
a. Antagonism
b. Additive
c. Synergism
d. Potentiation

24. An antagonist that occupies a receptor and the


effect produce cannot be overcome even if you
increase the concentration of the agonist; binding by
covalent bonds:
a. Reversible antagonist
b. Irreversible antagonist
c. Chemical antagonist
d. Physiologic antagonist

25. PHARMACODYNAMICS is associated with the


following:
a. Volume of distribution
b. Half-life
c. Clearance
d. Potency

11
MD2024 | PLATINGS

PHARMACOLOGY
USE AT YOUR OWN RISK, BESTIE

BACTERIAL MORPHOLOGY, STRUCTURE, 6. Which of the following is NOT part of the bacterial
CLASSFICIATION cell envelope?
1. A 67-year-old male diabetic admitted at the ICU was A. Peptidoglycan
diagnosed with septic shock. This complication is B. Plasma membrane
attributable to which of the following bacterial C. Cytoplasm
components? D. Capsule
A. Peptidoglycan
B. Core polysaccharide 7. Which of the following structures is not likely to be
C. O antigen immunogenic?
D. Lipid A A. Cytoplasm
B. Flagella
2. Which of the following possess antigenic property? C. Capsule
A. Capsule D. Pili
B. Peptidoglycan
C. Spore 8. Which of the following is NOT a component of
D. Cytoplasmic membrane endospores?
A. Chromosomes
3. A 23-year-old laborer presents with fever, B. Diaminopimelic acid
hepatomegaly and icteric sclerae days after wading in C. Keratin coat
floor waters. A diagnosis of leptospirosis caused by D. Small amount of moisture
Leptospira interrogans was made. Which of the
following BEST describe this causative agent? 9. Bacterial spores enable environmental survival
A. Very fine irregular coils against the following EXCEPT
B. Very fine regular coils with hook on one or both ends A. Desiccation
C. Course irregular coils B. Radiation
D. Fine regular coils with tapered ends C. Heat
D. Antibiotics
4. A 10-year-old boy was seen by a pediatrician
because of carbuncle on his right buttocks. A gram 10. Which of the following is a feature of fimbriae?
stain of the discharge showed Gram positive cocci in A. Mediates bacterial conjugation
clusters. Which of the following statements is correct B. Antiphagocytic
regarding this bacterium? C. Target of antibiotics
A. Its Gram stain reaction is due to the mycolic acid in the D. Twitching motion
cell wall
B. The cell wall is primarily composed of liposaccharide METABOLISM/PHYSIOLOGY/GROWTH AND
C. Has multi-layered peptidoglycan in its cell wall NUTRITION
D. Its outer membrane is made up of lipoligosaccharide 1. Which phase in the bacterial growth curve is
associated with beginning spore formation?
5. Which of the following is correct regarding lipo- A. Decline phase
oligosaccharides? B. Stationary phase
A. Lacks the long linear polysaccharide units C. Log phase
B. Possess O antigen D. Lag phase
C. Toxic moiety of the outer membrane
D. Responsible for resistance to adverse environmental 2. Which of the following is the major mechanism for
conditions. generation of ATP of facultative anaerobes?
A. Pentose Phosphate Shunt
B. Embden-Meyerhof- Parnas Pathway
C. Tricarboxylic acid Pathway
D. Entner Doudoroff Pathway

1
10. K.G., a 32-year-old female with chronic cough,
3. Bacteroides fragilis, based on their oxygen anorexia and recurrent back pain was diagnosed with
requirement is classified as which of the following? Pulmonary Tuberculosis Category I. The causative
A. Microaerophilic agent is an obligate aerobe. Which metabolic pathway
B. Facultative anaerobe will it utilize to obtain energy?
C. Aerobe A. Embden-Meyerhof-Parnas Pathway
D. Obligate anaerobe B. Entner Doudoroff Pathway
C. Tricarboxylic acid Pathway
4. Autotrophs obtain their carbon source from which D. Pentose Phosphate Shunt
of the following?
A. Oxygen STERILIZATION AND DISINFECTION
B. Glucose 1. The following disinfectants act by disrupting the
C. Nitrogen plasma membrane EXCEPT:
D. Organic salts A. Chlorhexidine
E. Carbon dioxide B. Hypochlorite
C. Benzalkonium chloride
5. Which environmental requirement is referred to if D. Cresol
an organism is a mesophile?
A. pH 2. The laboratory scientist needs to prepare and
B. Osmotic pressure sterilize the enrichment culture medium for the day’s
C. Carbon dioxide work. Which procedure/agent is required for this
D. Temperature task?
A. Ethylene oxide
6. Carbon dioxide is required by chemoorganotrophs B. HEPA
for CO2 fixation and for the synthesis of which of the C. Autoclave
following? D. Hot air oven
A. Carbohydrates
B. Purines 3. Which of the following agents can be used to
C. Protein preserve food and drinks?
D. Fatty acids A. Ethyl alcohol
B. Aldehydes
7. Results of the stool culture of a patient with acute C. Propionic acid
gastroenteritis showed growth of a halophilic D. Mercurial
organism. Which of the following should be in high
concentration for optimal growth of this organism? 4. To use a hot air oven for sterilization, the
A. Osmotic pressure temperature and time should be set to which of the
B. Temperature following?
C. Salt A. 1210C for 15 minutes at 15 psi
D. pH B. 1000C for 30 minutes
C. 1700C for 2 hours
8. Which of the following organisms utilizes the D. 1600C for 2 hours
Tricarboxylic acid Pathway?
A. Microaerophilic 5. To make sure that sterilization using the autoclave is
B. Obligate Aerobes adequate, a special tape is attached to the materials to
C. Aerotolerant be sterilized. What is the indicator present in the tape?
D. Obligate Anaerobes A. Bacillus stearothermophilus
B. Methylene blue
9. Which of the following inorganic ions stabilizes C. Clostridium tetani
ribosomes? D. Malachite green
A. Magnesium
B. Iron 6. Which of the following is true of betadine?
C. Potassium A. Denatures protein
D. Calcium B. Removes dirt through the process of emulsification
C. Binds to tyrosine residues in proteins
D. Functions best at an alkaline pH

2
7. Which of the following can physically remove 4. Which of the following DOES NOT eliminate non-
microbes from air? resident microorganisms from the skin?
A. Ultraviolet rays A. Alkaline pH of sweat
B. Sunlight B. Presence of lysozyme
C. Membrane filter C. Constant sloughing of the epidermis
D. HEPA filter D. Fatty acids in sebaceous secretions

8. Which of the following is involved in microwave 5. Which of the following is the chemical composition
ovens? of the component of endotoxin that is responsible for
A. Osmosis its effects on the patient?
B. Radiation A. Lipopolysaccharide
C. Dry heat B. Amino acid
D. Moist heat C. Polypeptides
D. Fatty acid
9. Which of the following is the process by which soaps
remove dirt? 6. Which of the following is a characteristic of
A. Dehydration exotoxin?
B. Purification A. Present in Gram negative bacteria only
C. Saponification B. Closely bound to cell wall
D. Emulsification C. Heat stable
D. Heat labile
10. Which of the following forms of alcohol is used as a
preservative for products like Cetaphil? 7. Which of the following is synthesized by normal
A. Benzyl alcohol intestinal flora?
B. Denatured alcohol A. Vitamin A
C. Isopropyl alcohol B. Vitamin K
D. Ethyl alcohol C. Vitamin C
D. Vitamin E
NORMAL FLORA/ BACTERIA & DISEASE
1. Which of the following organisms are dominant in 8. Which of the following members of the normal flora
the large intestines? in the genitourinary tract is reduced in menopaused
A. Aerobes women?
B. Anaerobes A. Staphylococcus aureus
C. Microaerophiles B. Enterococci
D. Aerotolerant C. Streptococcus viridans
D. Lactobacilli
2. Which of the following refers to virulence of an
organism? 9. Which of the following is an example of a nosocomial
A. Degree of pathogenicity infection?
B. Reproduction in high salt concentration A. 62-year-old diabetic female developed abscess at the
C. Growth under aerobic conditions lower back
D. Ability to produce disease B. 32-year-old male student developed colds two days
after attending a party
3. Physical examination performed on an C. 82-year-old female developed sepsis due to ruptured
asymptomatic 42-year-old male seaman revealed diverticulum
essentially normal findings. His hepatitis profile was D. 78-year-old stroke patient confined in the ICU with an
positive for Hepatitis B surface antigen. The patient in-dwelling catheter developed UTI
can be considered as which of the following based on
the result of his PE and blood examination? BACTERIAL GENETICS
A. Contact 1. Which of the following is a feature of prokaryotic
B. Suspect genome?
C. Subclinical A. 2 or more linear chromosomes
D. Carrier B. Structure maintained by histones
C. With nucleoid not enclosed by a membrane
D. Diploid

3
2. If an incorrect amino acid is inserted into the 9. Which of the following IS NOT true of DNA
chromosome leading to production of a structure?
malfunctioning protein, what type of mutation has A. Double helix
occurred? B. Backbone consists of phosphate and deoxyribose
A. Missense mutation C. Consist of 2 complementary strands of the same
B. Non-sense mutation polarity
C. Silent mutation D. Composed of four heterocyclic bases
D. Point mutation
10. Which of the following is true of the process of
3. According to the Central Dogma, which of the translation?
following represents the flow of genetic information in A. Occurs in the endoplasmic reticulum
cells? B. The A site is the binding site for Trna
A. DNA to protein to RNA C. mRNA information is converted into protein
B. Protein to DNA to RNA D. enzyme involved is DNA polymerase
C. RNA to DNA to protein
D. DNA to RNA to protein INTRO TO IMMUNULOGY
1. The movement of neutrophils across the
4. What would be the effect of a nonsense mutation on endothelium to the infected site under the influence of
the protein product? external chemical signals initially starts with which of
A. No effect on protein product the following?
B. Different protein produced A. Phagocytosis of pathogens or particulate antigens in
C. functioning protein the circulation
D. Production of truncated protein B. The release of histamine from basophils resulting to
localized vasodilation
5. Which of the following mutagens act by producing C. Attachment of neutrophils to adhesion molecules on
free radicals that cause single strand breaks in the endothelial cells
DNA? D. Macrophages releasing cytokines that attracts
A. Ionizing radiation neutrophils
B. Alkylating agents
C. Ultraviolet light 2. Which of the following substances involves
D. Heat processing by proteasomes prior to its delivery to the
surface for presentation to CD8+ cells?
6. Which type of mutation occurs when a portion of a A. T-cell receptor proteins
chromosome is left out? B. Superantigens
A. Inversion C. Exogenous antigens
B. Deletion D. Endogenous antigens
C. Translocation
D. Insertion 3. Which of the following refers to the ability of the
immune system to recognize self-antigens from non-
7. Mutations occurring in bacteria self-antigens?
A. are seen only in Gram-positive organisms A. Hypersensitivity
B. are always induced by mutagens and never B. Tolerance
spontaneous C. Specific immunity
C. cannot be passed on to progeny D. Antigenic immunity
D. may sometimes be beneficial to them
4. Which among the following is the most potent
8. Which of the following enzymes utilized during DNA antigen?
replication functions to relieve torsional strain A. Amino acid homopolymers
produced during unwinding? B. Complex proteins
A. Helicase C. Carbohydrates
B. Primase D. Lipids and monosaccharides
C. Ligase
D. Topoisomerase

4
5. A large carrier protein is needed by which of the 2. The formation of C3Bb is inhibited by which of the
following substances to induce an immune response? following?
A. Virus A. Complement receptor-1
B. Antigen B. Factor P
C. Hapten C. Decay accelerating factor
D. Antibody D. Factor H

6. Intracellular pathogens or virus-infected cells are 3. Properdin deficiency is associated with which of the
recognized and destroyed by which cellular following conditions?
component of the innate immune system? A. Hemolytic uremic syndrome
A. Natural Killer cells B. Severe Neisseria bacteremia
B. Activated macrophages C. Meningococcal infection
C. CD8+ Cytotoxic cells D. Pneumococcal septicemia
D. Neutrophils
4. Which of the following complement components are
7. Which of the following immunoregulatory proteins NOT required in the alternative pathway?
induces a viral resistant state to neighboring cells by A. C8, C9
activating its gene to inhibit viral replication? B. C5, C6, C7
A. Cytokines C. C3bBb
B. Complement protein D. C1, C2, C4
C. Defensins
D. Interferons 5. Which of the following complement components
function to lyse microorganisms?
8. The effective and specific killing of intracellular A. C5-C9
pathogens is best carried out by which particular B. C3-C5
component of the adaptive immune system? C. C1-C9
A. Natural killer cells D. C8-C9
B. Complement system
C. T- cytolytic cells 6. In the final step of complement activation, which
D. Antibodies complement protein polymerizes to form perforin-like
structures stabilizing the membrane attack complex?
9. Which of the following agents is used to augment A. C7
the effects of a vaccine by stimulating the immune B. C8
system to respond more vigorously providing C. C9
increased immunity to a particular disease? D. C6
A. Epitope
B. Adjuvant 7. _____ is the complement protein that recruits
C. Antigen inflammatory cells while _____ is the complement
D. Immunogen component which facilitates phagocytosis.
A. C4a/C3a
10. Which of the following statements is true B. C5a/C5b
regarding natural killer cells? C. C5b/ C3a
A. Circulate in the blood and mature into macrophages D. C3a/C3b
B. Contribute to innate immunity by providing protection
against viruses 8. Engulfment and killing of organisms by phagocytic
C. Store enzymes and toxic proteins in granules that can cells are facilitated through the help of which
be released upon activation of the cells complement component?
D. Destroy pathogens within intracellular vesicles A. C5b-C9
B. C5a
COMPLEMENT SYSTEM C. C3bBb
1. Which of the following cleaves C3 to C3a + C3b in D. C3b
the MBL pathway?
A. C4b2a
B. C3bBbC3b
C. C4b2b
D. C3bBb

5
9. In the Alternate pathway, which of the following 5. The intracellular signal initiated by antigen binding
serves as C3 convertase? to the T-cell receptor is generated by which set of
A. C4b2b3b molecules expressed on the T cell membrane?
B. C4aC2b A. CD4
C. C4b2a B. CD28
D. C3bBb C. CD45
D. CD3
10. A 3-week-old neonate was admitted due to sudden
onset of high-grade fever and seizure episodes. In the 6. T cell receptor
ER, the attending physician noted bulging fontanelles, A. has 4 chains
with lateral gaze. The clinical impression was B. recognizes peptide-MHC complex
meningitis. Short, gram-positive rods singly and in C. surface IgM
chains were isolated from the CSF and identified as D. recognizes free antigen
Listeria monocytogenes. This organism contains
specific carbohydrate moieties on its surface which 7. CD4 T cell that enhances neutrophil activity
will activate what complement system pathway? A. NK cell
A. MBL Pathway B. TH17
B. Properdin and Classical C. T Reg cell
C. Classical Pathway D. TH17
D. Alternative Pathway
8. Pattern recognition receptor EXCEPT
CELL MEDIATED IMMUNITY A. interferon
1. What are the two signaling T helper cell-derived B. NOD receptor
molecules responsible for inducing class switching C. RIG-I
from IgM to IgG? D. TLR-4
A. B7 and IL-7
B. CD34 and IL-4 9. Toll-like receptors are found on which of the
C. CD40 ligand and IFN-γ following immune antigen presenting cells?
D. CD20 and IL-2 A. T
B. NK
2. Which of the following cytokines enhances C. Dendritic
macrophage intracellular killing? D. B
A. Interleukin-3
B. Interferon gamma 10. Which of the following immune cells utilize
C. Interferon alpha perforin-granzyme system to kill pathogens?
D. Interleukin-6 A. Macrophages
B. B lymphocytes
3. What glycoprotein on virally infected cells provides C. Dendritic cells
a target for cell-mediated cytotoxicity? D. Natural killer cells
A. MHC class II
B. MHC class I
C. CD4
D. CD8

4. Toll-like receptors are found on which of the


following immune cells?
A. T lymphocytes
B. Dendritic cells
C. B lymphocytes
D. NK cells

6
MD2024 | PLATINGS WITH (SOME) RATIO

PHARMACOLOGY - MIDTERMS
USE AT YOUR OWN RISK

TOPICS Due to inhibition of cyclooxygenase pathway, inhibiting


NSAIDS Quiz (no plating for this topic) prostaglandin and thromboxane synthesis
Blood Quiz
Blood Plating
Dyslipidemia Quiz Tocolysis: There are some drugs with tocolytic activity,
Dyslipidemia Plating observe this in
Antimicrobials I Quiz NSAIDs with indole derivatives, such as Sulindac and
Antimicrobials I Plating Indomethacin
Antimicrobials II Quiz
Antimicrobials II Plating
3. NSAID bioavailability is not changed by food intake:
a. Naproxen
NSAIDS POST-TEST
b. Fenoprofen
1. The following are non-selective COX inhibitors, c. Oxaprozin
except d. Paracetamol
a. Piroxicam - non
b. Ibuprofen
NSAIDs are weak organic acids, except for
c. Mefenamic Acid
Nabumetone. They are rapidly absorbed in GIT, but
d. Meloxicam
there are two drugs whose bioavailability are not
affected by food: Fenoprofen and Piroxicam
Meloxicam is a selective COX-2 inhibitor. This includes
celecoxib, valdecoxib, rofecoxib, parecoxib, etc.
4. The ff are TNF-alpha, except:
REMEMBER: -coxib = selective COX-2 inhibitors
a. Infliximab
b. Tocilizumab
Piroxicam, Ibuprofen, and Mefenamic are non-
c. Etanercept
selective. They both act on COX-1 and COX-2
d. Adalimumab
Acetaminophens and Paracetamol are not NSAIDS.
These drugs are known as DMARDs (Disease Modifying
They are para-amino phenols.
Anti-Rheumatic Drugs)
Aspirin, Sodium Salicylate, Salsalate, Diflunisal are
Conventional DMARDs:
salicylic acid derivatives, so they inhibit irreversibly
Methothrexate Azathioprine
prostaglandin synthesis
Cyclophosphamide Cyclosporine
Leflunomide Sulfasalazine
Indomethacin and Sulindac are para-chlorobenzoic
Mycophenolate mofetil
acid derivatives or indoles
Chloroquine & Hydrochloroquine (anti-malarial
effect)
Phenylbutazone and Azapropazone are Pyrazalone
derivatives.
Biologic DMARDs:
TNF-alpha Infliximab Etanercept
2. The ff are major effects of NSAIDS, except:
a. Anti-inflammatory Neutralizing Adalimumab Certolizumab
b. Analgesic Agents Golimumab
c. Tocolytic Anakinra
IL-1 Neutralizing
d. Antipyretic Canakinumab
Agents
Rilonacept
NSAIDs have 4 major effects: Depletes B-cells Rituximab
Anti-inflammatory Interferes w/ T cell Abatacept
Antipyretic activation
Analgesic Anti-IL-6 Receptor Tocilizumab
Anti-platelet Antibody

1
5. Which of the ff is correct regarding DMARDs 9. Short acting beta 2 agonists are administered via:
a. Might arrest or slow the progression of bone in a. All are correct
cartilage destruction b. Metered dose inhaler
b. Effects may take 6 weeks to 6 mos to become evident c. Nebulizers
c. Exert minimal direct non-specific anti-inflammatory d. Oral administration
analgesic effect
d. All the above 10. MT is a 10-y/o boy who is bought into the ED by his
6. This drug is used in gouty arthritis to relieve pain by parents, diagnosed with a bronchial asthma attach.
inhibiting leukocyte migration and phagocytosis:
a. Colchicine aggressive conventional therapy with short acting
beta 2 agonist and systemic corticosteroids. He is
The action of Colchicine is to reduce or inhibit the admitted to the hospital and placed in the ICU. It is
migration; its effect is on the microtubules. It will decided to try a trial dose of Theophylline, which has a
relieve the pain by binding to your tubulin dimers, narrow therapeutic window of 5-15 ug/ml. what toxic
preventing the polymerization of microtubules. Walang
microtubule, walang locomotion theophylline levels become too high?

7. The newest urate-lowering therapy for refractory a. Sedation


gout which is contraindicated in patient with: b. Seizures and arrythmias
a. Gastritis c. Bradycardia
b. Heart failure d. Hyperkalemia
c. G6PD deficiency
d. Porphyria Sa CVS, ang pinaka common is arrythmia.
Sa CNS, seizure.
Uricosuric agents are contraindicated in patients with
G6PD deficiency. CLINICAL CASE: NSAID POST TEST
1. Drugs of choice for producing a rapid relief from
8. Which is most potent and effective controller bronchoconstriction:
medication of asthma? a. Inhaled corticosteroids
a. Intake of anti-muscarinic drugs b. Inhaled anti-muscarinic agents
b. Inhaled beta 2 agonists c. Inhaled cromolyn sodium
c. Inhaled corticosteroids d. Inhaled short acting beta 2 agonist
d. All are correct
Bronchodilators = Inhaled short acting beta agonist
Dalawa kasi yun, controller and reliever.
Controller = Anti-inflammatory corticosteroids 2. Possible trigger in the case of FDM:
Reliever = Bronchodilators a. Respiratory infections
b. Specific allergen
Since the question is controller, the answer must be an c. Extreme temperature
anti-inflammatory agent, such as inhalation d. Strong emotions
corticosteroids:
Beclometasone Budesonide Ciclesonide Respiratory infection because patient had history of
Flunisolide Mometasone Triamcinolone bronchitis.

If reliever, release inhibitors yan:


Cromolyn Sodium Nedocromil a. Moderate to severe respiratory distress with bilateral
expiratory wheezing
or Leukotriene Antagonists: b. Mild respiratory distress with bilateral expiratory
Zeliuton Montelukast Zafirlukast wheezes
c. Moderate to severe respiratory distress with bilateral
Anti-inflammatory agents that will relieve inflammation inspiratory wheezing
= CONTROLLER d. Severe respiratory distress with bilateral inspiratory
wheezes
Anti-inflammatory agents that will provide rapid relief
= BRONCHODILATOR (short-acting drugs)

2
4. Antidote for acetaminophen/paracetamol overdose: 10. This is an effective treatment to prevent recurrent
a. All are correct gouty flares, but is not for use during acute flares because
b. N-acetyl cysteine as monotherapy it makes the symptoms worse?
c. Mucomyst a. Allopurinol
d. Oral methionine
QUIZ: BLOOD
Oral methionine is just as effective as N-acetylcysteine 1. What is most likely the parasite that caused the anemia?
a. Pinworm
as an antidote. Mucomyst is also an N-acetyl cysteine
b. Ascaris
c. Hookworm
5. Minor but highly active metabolites/s of d. Tapeworm
acetaminophen/paracetamol responsible for liver and
kidney toxicity: 2. Microcytic hypochromic anemia
a. Glucuronide a. Iron deficiency
b. Folic acid
b. Acetyl-p-benzoquinone c. All of the above
c. Glucuronide and acetaminophen SO4 d. Vit. B12 deficiency
d. Acetaminophen SO4
Folic and Vit B12 are megaloblastic so they have large RBC.
6. What contributes to further depletion of hepatic
glutathione reserve in patient FDM? 3. Heparin antagonist
a. Protamine sulfate
a. Intake of other NSAIDS
b. All of the above
b. Infection c. Vit K
c. Alcohol d. Warfarin
d. Diabetes
Given if you have side effects already of heparin.
Alcohol depletes glutathione.
4. Inhibitor of fibrinolysis (pro-coagulants)
a. Aminocaproic acid
If chronic, alcohol induces the activity of P450 b. Both aminocaproic acid and tranexamic acid
c. Tranexamic acid
7. What is the complaint of FDM with regard to her acute
gouty arthritis? Pro-coagulants are given to patients with bleeding tendencies.
a. Pain on the big toe, right
5. Inhibitors of clot formation
b. Acute pain in her right elbow
a. Dicumarol
c. Back pain b. Warfarin
d. Acute intense pain c. Heparin
d. All of the above
8. A purine oxidase inhibitor taken by FM for her gouty
flare prevention. Heparin has anti-thrombin property and this facilitates the
activity of anti-thrombin. It is also a fibrinolytic agent.
a. Febuxostat
b. Allopurinol Warfarin and Dicumarol are inhibitors of Vit K, which can also
lead to fibrinolysis.
Febuxosat is also a xanthine inhibitor, but the question
is the one taken by the patient, which is Allopurinol 6. Causes platelet aggregation
a. Adenosine diphosphate
b. Both adenosine diphosphate and thromboxane
9. Considered the first line of therapy for acute gouty c. Thromboxane
flares:
a. Colchicine Both adenosine diphosphate and thromboxane are platelet
b. NSAIDS aggregators. Prostacyclin is anti-platelet aggregator.
c. Methotrexate
7. In case 3 where the patient is a known vegetarian and
d. Aspirin low dose
eventually showed signs of glossitis and numbness and pallor,
what is the most likely diagnosis?
Kapag acute gout flare, the first line of therapy is a. Macrocytic anemia
NSAID because you have to relieve the pain and b. Both macrocytic anemia and megaloblastic anemia
inflammation. c. Megaloblastic anemia

Case 3 = Vit. B12 anemia


Aspirin, no. You can also give Colchicine, but it is not
the first line, but you still give it together with NSAID

3
8. What is the vitamin that is deficient in this case? 16. Because of chronic blood loss caused by the blood sucking
a. Vitamin B12 parasite, what element is being lost by the patient leading to
b. Vit A deficiency?
c. Vit D a. Copper
d. Vit C b. Zinc
c. Iron
9. What injectable anticoagulant should be given IV to prevent d. Oxygen
additional clot formation to prevent this case from getting
worse? Element present in hemoglobin is Iron
a. Heparin
b. Dicumarol 17. Protease inhibitor that inactivates coagulation away from
c. Warfarin the site of injury
d. Warfarin and Dicumarol a. Antitrypsin
b. Both antitrypsin and macroglobulin
10.What are the side effects of heparin? c. Macroglobulin
a. Bleeding
b. Osteoporosis 18. Used to reverse the effect of warfarin
c. Hair loss A. Vit K
d. All of the above b. Both Vit K and protamine sulfate
c. Protamine sulfate
11. What are the different forms of this element are available for
therapeutic use? Warfarin inhibits Vit K, so substance used to reverse its effect is
a. Ferrous fumarate Vit K. Protamine sulfate is for heparin
b. Ferrous sulfate
c. Ferrous gluconate 19. The following are activators of plasminogen to plasmin which
d. All of the above will lead to fibrinolysis
a. All of the above
12. Dissolves the clot b. Urokinase
a. Fibrin c. Streptokinase
b. Plasmin d. Anistreplase
c. Both plasmin and fibrin
20. Promotes activation of plasminogen to plasmin
Plasmin activators activate plasminogen to plasmin, so plasmin is a. Anistreplase
the one that dissolves the clot, not fibrin, because fibrin is the b. Both anistreplase and streptokinase
clot itself. c. Streptokinase

13. What is the antagonist for heparin? Anistreplase, streptokinase, and urokinase are examples. They
a. Warfarin are fibrinolytics so they inhibit clot formation.
b. Vit K
c. All of the above PLATINGS: BLOOD
d. Protamine sulfate
1. Inhibitors of clot formation which indirectly block anti-
thrombin
14. What other vitamin could be possibly result to the same
a. dicumarol
deficiency symptoms?
b. protamine sulfate
a. Folic acid
c. heparin - facilitates the activity of anti-thrombin
b. Vit E
d. warfarin
c. Thiamin
d. Ascorbic acid
2. Causes platelet aggregation
It is given in the differential diagnosis. Since Vit B12 and Folic acid
a. streptokinase
are both needed in the maturation of RBC (conversion from
b. thromboxane platelet aggregator
reticulocyte to mature RBC), without these two, the RBC are not
c. urokinase
mature, they are megaloblastic (big sized).
d. aminocaproic acid
15. In case 2 where the patient showed signs of anemia, what is 3. Splenic rupture is a rare but serious complication of the use of
the most likely diagnosis? this myeloid growth factor for PBSC mobilization
a. Both microcytic anemia and hypochromic anemia a. G-CSF
b. Microcytic anemia b. interleukin
c. Hypochromic anemia c. folate
d. deferoxamine
In case 2, we are dealing with iron deficiency anemia. So, it will
show microcytic (small-sized) and hypochromic (paler in 4. This drug does not activate plasminogen to plasmin and will
appearance) RBC not lead to fibrinolysis
a. urokinase
b. streptokinase
c. anistreplase
d. phosdiuesterase
First three choices enhance plasminogen activators.

4
5. Possible treatment for both hemophilia A and B 15. Approved for the secondary prevention of
a. factor IX thrombocytopenia in patients receiving cytotoxic
b. recombinant factor VIIa chemotherapy for treatment of non-myeloid cancers
c. factor VIII a. cyanocobalamin
d. all b. interleukin II
c. folacin
6. Heparin antagonist d. tetrahydrofolate
a. warfarin
b. protamine sulfate 16. The introduction of this drug represents a milestone in the
c. vit C treatment of chemotherapy-induced neutropenia
d. vit K a. Interleukin
b. G-CSF
7. Used to reverse the effect of warfarin c. Thrombopoietin receptor agonist
a. ADP d. Deferoxamine
b. Vit K
c. thromboxane 17. A specific, irreversible thrombin inhibitor from leech saliva
d. protamine sulfate a. Edoxaban
b. Hirudin
Warfarin antagonizes vitamin K, so if you want to reverse its c. Bivalirudin
effect, you use vitamin K d. Argatroban

8. Inhibit GP IIb IIIa, EXCEPT 18. Protease inhibitor that inactivates coagulation proteins away
a. Abciximab from the site of injury
b. Ticlopidine a. Antitrypsin
c. Tirofiban b. Thromboxane
d. Eptifibatide c. Aminocaproic Acid
d. Glycoprotein IIb
9. This is also known as the Christmas factor and is affected by
Warfarin 19. The following is/are given parenterally, except:
a. factor X a. Argatroban
b. factor VIII b. Lepuridin
c. factor IX c. Dabigatran
d. factor XI d. Bivalirudin

10. This drug is an oral direct factor Xa inhibitor that is approved 20. The following are hematopoietic growth factor except
for use in the prevention of stroke in nonvalvular atrial a. deferasirox
fibrillation b. thrombopoietin receptor agonist
a. Argatroban c. interleukin
b. Bivalirudin d. epoetin alpha
c. Apixaban
d. Lepirudin 21. This drug inhibits activated factor X in the final common
pathway of clotting has a rapid onset of action and short half-
11. All but one is a megakaryocyte growth factor life
a. Etrombopag a. hirudin
b. Pegfilgrastim b. rivaroxaban
c. Romiplostim c. heparin
d. Interleukin 11 d. warfarin
12. All about Apixaban, except
a. Inhibits factor Xa 22. This drug induces release of reticulocytes from bone marrow
b. Rapid onset of action Erythropoietin
c. Prevention of stroke in nonvalvular atrial fibrillation
d. Given intravenously 23. The following drugs reduce platelet aggregation by
inhibiting ADP pathway, except:
13. This is the major adverse effect of heparin a. Clopidrogel
a. bleeding heparin is a fibrinolytic agent b. Prasugrel
b. mineralocorticoid deficiency c. Aspirin
c. alopecia
d. osteoporosis 24. The following is true about Vit K, except
a. antidote of warfarin
14. This anticoagulant accelerates the effects of antithrombin b. vitamin K2 in food
on factors IIa, IXa, and Xa by 1000-fold c. there is a need to give vit k to newborn babies
a. heparin d. vit. K1 is available clinically in oral and parenteral form
b. hirudin
c. apixaban
d. warfarin

5
QUIZ: DYSLIPIDEMIA c. cholestyramine
1. It is classified as high-intensity statin d. fibrates
a. Simvastatin
b. Atorvastatin 10mg 11. Which of the following is true regarding the mechanism of
c. Lovastatin action of HMG-CoA reductase inhibitor?
d. Rosuvastatin 20-40mg a. Induce an increase in high affinity LDL receptors
b. Decreases the fractional catabolic rate of LDL
Simvastatin and Lovastatin are not part of it c. Inhibits the last step in sterol biosynthesis
d. Decreases the hepatic extraction of LDL precursors
Atorvastatin should be 40-80mg
Ang iniinhibit ng HMG-CoA reductase is the rate-limiting step in
2. Which of the following is a modifiable risk factor? cholesterol synthesis
a. smoking
b. age 12. All but one statin should be given at night because
c. sex cholesterol synthesis occurs predominantly at night:
d. ethnicity a. Pravastatin
b. Rosuvastatin
3. It is the most atherogenic lipoprotein c. Simvastatin
c. LDL-C d. Fluvastatin

4. It is the target LDL-C in patients with hypertension and 13. Absorption of this statin is affected by the presence of food
comorbidities? a. rosuvastatin
<100mg/dL b. lovastatin
c. atorvastatin
5. It is the target HDL-C to improve cardiovascular outcome: d. pravastatin
a. 50 mg/dl
b. 40 mg/dl 14. The catabolism of this drug proceeds chiefly through
c. <40 mg/dl CYP2C9:
d. >50 mg/dl a. rosuvastatin
b. atorvastatin
6. It is one of the important side effects of statins that needs to c. simvastatin
emphasize in the patient d. lovastatin
a. dermatitis
b. gastritis 15. It functions primarily as ligands for the nuclear transcription
c. rhabdomyolysis receptor PPAR-a:
d. arthritis a. coleveselam
b. fibrates
An important side effect of statin is myopathy, and the most c. statins
common is rhabdomyolysis. You have to monitor this by d. niacin
monitoring your CK. One of the manifestations is tea-colored
urine. 16. It inhibits VLDL secretion thus decreasing the production of
LDL and increased clearance of VLDL via the LPL pathway:
7. A statin that can be taken anytime of the day a. coleveselam
a. lovastatin b. ezetimibe
b. pravastatin c. niacin
c. rosuvastatin d. fibrates
d. simvastatin
17. Which of the following is true regarding high density
8. Which of the following is an acute fatal complication of severe lipoprotein (HDL)?
hypertriglyceridemia? a. causes an impaired cholesterol retrieval
a. rhabdomyolysis b. stimulates oxidation of atherogenic lipoproteins - LDL
b. pancreatitis c. helps reduce the levels of atherogenic lipoproteins
c. myositis d. promotes deposition of cholesterol in artery wall
d. hepatitis
18. It is formed in the intestine and carry triglycerides of dietary
origin:
All can be a complication, but the most fatal is pancreatitis.
a. chylomicrons
b. VLDL
9. Which of the following has the highest source of triglycerides
c. IDL
in the diet?
d. LDL
a. rice
b. meat
19. It is secreted by the liver and exports triglycerides to
c. alcohol
peripheral tissues:
d. legumes
a. IDL
b. LDL
10. It has the greater reduction in the level of triglycerides
c. HDL
a. niacin
d. VLDL
b. statin

6
20. It is an important component of metabolic syndrome aside 9. It participates in the retrieval of cholesterol from the artery
from insulin resistance, hypertension, and abdominal obesity: wall and inhibit oxidation of atherogenic lipoproteins
a. hypertriglyceridemia a. VLDL
b. anemia b. HDL good cholesterol
c. polycythemia c. Triglycerides
d. uremia d. LDL

PLATING: DYSLIPIDEMIA 10. A 57 y/o male patient is on atorvastatin for almost 2 years.
1. Treatment of heterozygous familial hypercholesterolemia in You should temporarily discontinue this drug in the event of the
children, EXCEPT: following:
a. May be treated with a resin or reductase inhibitor a. Trauma
b. Treatment usually starts before 7 or 8 years of age b. Major surgery
c. The decision to treat a child should be based on the level of LDL, c. Serious illness
d. All choices are correct
d. Treatment starts when myelination of the central nervous system
is essentially complete 11. True of Homocysteine
a. Reduction of high levels of homocysteine is especially important
2. This is the fibrate of choice for use in combination with a in individuals with elevated levels of Lp(a)
statin b. Initiates proatherogenic changes in endothelium
a. Fenofibrate c. All are correct
b. Gemfibroxil d. Can be reduced in many patients by restriction of total protein
c. Clofibrate intake
d. All of the choices are correct
12. Which of the following drugs is not part of the combination
3. The following is/are TRUE statements on DIET in managing treatment which acts in a complementary fashion to normalize
dyslipidemia cholesterol in patients with severe disorders involving elevated
a. All are correct LDL?
b. Total fat, sucrose, and especially fructose increases VLDL a. Ezetimibe
c. Alcohol can cause significant hypertriglyceridemia increasing b. Resins
hepatic secretion of VLDL c. Niacin
d. Cholesterol and saturated and trans-fats are the principal factors d. Fibrates
that increase LDL e. Reductase inhibitors

4. A PCSK9 Humanized Monoclonal Antibody which inhibits 13. Incorrect statement/s of LDL Receptor Deficient Familial
catabolism of LDL receptor Hypercholesterolemia (FH)
a. Ezetimibe a. This is an autosomal dominant trait
b. Evolocumab b. Tendon xanthomas are often present
c. Mipomersen c. Coronary disease tends to occur prematurely
d. Lomitapide d. Triglycerides are usually abnormal too

5. The combination is sometimes useful in treating patients with 14. Which of the following statements is correct about drug
familial combined hyperlipidemia who are intolerant of niacin or combination in the treatment of dyslipidemia
statins but may increase the risk of cholelithisasis a. When LDL level is elevated and VLDL level is decreased initially
a. HMG-CoA Reductase Inhibitors + Bile Acid-Binding Resins b. When level of Lp(a) and HDL are both elevated
b. Reductase Inhibitors + Fibrates c. When VLDL levels are significantly increased during treatment of
c. Niacin & Reductase Inhibitors hypercholesterolemia with a resin
d. Fibric Acid Derivatives + Bile Acid-Binding Resins d. When LDL or VLDL levels are normalized with a single agent

6. Secondary causes of Hypertriglyceridemia 15. An antisense oligonucleotide that targets apo B-100, mainly
a. Alcohol ingestion in the liver:
b. Corticosteroid excess a. Lomitapide
c. All are correct b. Mipomersen
d. Hypothyroidism c. Alirocumab
d. Torcetrapib
7. TRUE regarding the mechanism of action of NIACIN:
a. All choices are correct 16. It denotes an increased level of triglyceride
b. Decreases the catabolic rate for HDL a. Hyperlipidemia
c. Inhibits VLDL secretion, in turn decreasing production of LDL b. Dyslipidemia
d. Reduces Fibrinogen levels c. Hyperlipoproteinemia
d. Hyperlipemia
8. It is a cellular component in atherosclerotic plaques which are
transformed macrophages smooth muscles with cholesteryl 17. The most effective agent for increasing HDL and the only
esters: agent that may reduce Lp(a)
a. Triglycerides a. Gemifibrozil
b. VLDL b. Probucol
c. Foam cells c. Niacin
d. LDL d. Atorvastatin

7
18. A 50 y/0 male with dyslipidemia was prescribed a drug for an 2. Which of the given choices is not correctly matched
elevated cholesterol. He developed constipation and bloating a. Lipopeptides-Daptomycin
after taking 15mg granules per day for a week. The drug taken is b. Cephalosporin-Cefotetan
possibly: c. Glycopeptides-Vancomycin
a. Fenofibrate d. Carbapenems-Telavancin
b. Ezetimibe
c. Colestipol 3. A group of cephalosporin with good body fluid and tissue
d. Any of the choices is correct penetration
a. First generation
19. This is added to statin therapy, providing additional effect, b. Second generation
decreasing LDL levels, and further reducing the dimensions of c. Third generation
atherosclerotic plaques 4. A second generation which can be combined with macrolide
a. Ezetimibe for the treatment of Community Acquired Pneumonia
b. Colestipol a. Loracarbef
c. Any of the choices is correct b. Ceftazidime
d. Fenofibrate c. Cefalexin
d. Cefuroxime
20. An MTP inhibitor but is currently restricted to patients with
homozygous familial hypercholesterolemia 5. Excessive level of this Carbapenems may lead to seizure in
a. Alirocumab patient with renal failure
b. Torcetrapib a. Imipenem
c. Mipomersen b. Meropenem
d. Lomitapide c. Doripenem
d. Eratpenem
21. Which of the following drugs is not part of the combination
treatment which acts in a complementary fashion to normalize 6. All but one is true about red man syndrome
cholesterol in patients with severe disorders involving elevated a. it is one of the adverse effects of aztreonam
LDL? b. due to the release of histamine
Fibrates c. it is an infusion related flushing
Resins d. can be pretreated with anti histamine
Reductase inhibitors
Niacin 7. The following choices are second generation Cephalosporin
Ezetimibe except
a. cefalexin first generation
22. Gemifibrozil 600 mg once daily or bid is recommended in the b. cefotetan
following c. cefoxitin
a. In treating the hypertriglyceridemia that results from treatment d. cefuroxime
with antiviral protease inhibitors
b. In hypertriglyceridemia in which VLDL predominate 8. A glycopeptide for the treatment of C. difficile infection
c. In dysbetalipoproteinemia a. teicoplanin
d. All choices are correct b. vancomycin
c. telavancin
23. It is the major clinical sequelae of hyperlipidemia d. oritavancin
Bleeding
Acute pancreatitis 9. Penicillin are poorly absorbed in the following organ except
Diabetes a. eyes
Cholelithiasis b. prostate
c. CNS
24. It is the major clinical sequelae of hyperlipidemia d. skin
a. cholelithiasis
b. acute pancreatis 10. This oral form of penicillin is indicated only in minor infection
c. diabetes because of poor bioavailability
d. bleeding a. Procaine Penicillin G
b. Nafcillin
25. It is secreted by the liver and exports triglycerides to c. Benzathine Penicillin G
peripheral tissues d. Penicillin V
a. VLDL
b. LDL 11. Available cephalosporin for surgical prophylaxis of acute
c. HDL appendicitis
a. Cefaclor
QUIZ: ANTIMICROBIALS I b. Cefmetazole
1. Correct statement about Cefazolin c. Cefazolin
a. A good antibiotic for meningitis d. Loracarbef
b. Belong to second generation Cephalosporin
c. The only first-generation parenteral Cephalosporin
d. Penetrates freely to CNS

8
12. Non-pharmacological management of cellulitis, except
a. properly covered skin lesion 19. Penicillin is not indicated in the following condition
b. keeping the area clean a. conjunctivitis
c. may apply hydrocortisone cream to lesion b. pneumonia
d. avoid scratching the lesion c. benign prostatic hypertrophy
d. meningitis
Hydrocortisone is a steroid, which decreases the effect of
antibiotic 20. C in CURB 65 risk assessment of pneumonia means
a. confusion
13. Component of curb criteria except b. capillary refill
a. urea level c. cardiac rate
b. blood pressure d. cardiac output
c. temperature level
d. respiratory rate
14. The following is indicated for a patient with acute
appendicitis except
a. cefuroxime
b. cefazoline drug of choice
c. clavulanic amoxicillin
d. fluconazole for fungal infection

15. Mechanism of action of Beta lactam antibiotic


a. inhibits protein synthesis by bind to 30S ribosome
b. inhibits folic acid synthesis
c. inhibits protein synthesis by binding to 50S ribosome
d. interferes bacteria cell wall synthesis

16. Cefuroxime is a
a. Fourth generation cephalosporin
b. Third generation cephalosporin
c. Second generation cephalosporin
d. First generation cephalosporin

PLATING: ANTIMICROBIALS 1

1. Vancomycin is usually given parenteral because it is poorly


absorbed in the GIT. It is only given orally for the treatment of:
a. Meningitis caused by a penicillin-resistant strain of pneumonia
b. Necrotizing pneumonia caused by Staphylococcus aureus
c. Endocarditis caused by methicillin susceptible strains
d. Colitis caused by clostridium difficile

2. Nausea, vomiting, and diarrhea are common adverse effects


of
Sulbactam
Vancomycin
Carbapenem
Telavancin

3. RP 28/F diagnosed with Pelvic Inflammatory Disease; she will


17. Oral antibiotic indicated for case 3 with mild localized benefit from this second-generation Cephalosporin used to
staphylococcal I infection treat mixed anaerobic infections
a. Procaine penicillin Cefotaxime
b. Cloxacillin Cefoxitin
c. Bentzathine penicillin Ceftazidime
d. Ceftazidime Cefuroxime

A, C, D are not correct because they are susceptible to beta 4. Prevention of peptidoglycan elongation and cross linking
lactamase enzyme resulting to inhibition of cell wall synthesis is a mechanism of
action of all of the following EXCEPT:
18. All but one is common adverse effect of penicillin and Telavancin
cephalosporin Carbapenem
a. nausea Vancomycin
b. abdominal discomfort Teicoplanin
c. allergy
d. cellulitis used for the treatment of cellulitis, not as an adverse
effect

9
5. Gastrointestinal absorption of this penicillin is erratic, 14. MR 30/F is scheduled for surgery, and she is given an
therefore it is not suitable for oral administration intravenous infusion of 1g of this first-generation parenteral
Nafcillin cephalosporin
Amoxicillin Cephalexin
Dicloxacillin Cefazolin
Ampicillin Cefoxitin
Ceftriaxone
6. This drug is usually used in the treatment of tuberculosis
caused by strains of Mycobacterium tuberculosis that are 15. This drug is not used in the management of pneumonia due
resistant to first-line agents to its inactivation by pulmonary surfactants
Fosfomycin Meropenem
Daptomycin Fosfomycin
Cycloserine Vancomycin
Bacitracin Daptomycin

7. This is the mechanism of action of daptomycin 16. Telavancin and teicoplanin are alike in that both;
a. Bind to cell membrane, causing depolarization and rapid cell death a. Commonly caused infusion-related flushing due to release of
b. Prevents bacterial cell wall synthesis by binding to and inhibiting histamine
cell wall transpeptidases b. Inhibit cell wall synthesis by binding to the D-Ala-D-Ala terminus
c. Inhibits cell wall synthesis by binding to the D-Ala-D-Ala terminus of the peptidoglycan
of nascent peptidoglycan c. Disrupt the bacterial cell membrane potential and increases
d. Inhibits the cytoplasmic enzyme enolpyruvate transferase membrane permeability
d. Are commonly used for the treatment of hospital acquired
8. A second generation cephalosporin pneumonia
Cephalexin
All are correct 17. All but one is true statement about penicillin
Cefizxime a. Should never be used for viral infection
Cefaclor b. Blood level of penicillin decreases with simultaneous
administration of probenecid
9. Which if the following is a beta lactamase inhibitor? c. Is available in oral and parenteral form
Ertapenem d. Oral penicillin should be given 1-2 hours before or after a meal
Sulbactam
Oritavancin 18. A third generation cephalosporin
Teicoplanin Ceftriaxone
Cefaclor
10. Adverse effect of cephalosporin Cefuroxime
All are correct Cephalexin
Anaphylaxis
Hemolytic anemia 19. The only third generation cephalosporin with useful activity
Granulocytopenia against P. aeruginosa
Ceftazidime
11. A first generation of cephalosporins Cefipime
Cephalotin Cephalexin
Cefoxitin Cefixime
Ceftazidime
Cefuroxime 20. This antibiotic is highly nephrotoxic and is only used as a topical
agent
12. Seizure observed in carbapenems is most likely due to Daptomycin
Doripenem Bacitracin
Meropenem Cycloserine
Ertapenem Fosfomycin
Imipenem
QUIZ: ANTIMICROBIALS 2
13. This is the mechanism of action of Fosfomycin 1. This drug blocks the 30S in protein synthesis
a. Prevents bacterial cell wall synthesis by binding to and inhibiting a. chloramphenicol
cell wall transpeptidases b. gentamycin
b. Inhibits the cytoplasmic enzyme enolpyruvate transferase c. clarithromycin
c. Binds to cell membrane, cousin depolarization and rapid cell death d. clindamycin
d. Inhibits cell wall synthesis by binding to the D-Ala-D-Ala terminus
of nascent peptidoglycan 2. The 50S inhibitor is a macrolide
a. erythromycin
b. netilmicin
c. gentamicin
d. tobramycin

10
3. These drugs are ototoxic and nephrotoxic
a. chloramphenicol 2. Minimally absorbed in the GIT thus is preferred to be given
b. trimethoprim sulfamethoxazole parenterally for systemic infection
c. aminoglycosides Cephalosporin
d. tetracycline Aminoglycoside
Erythromycin
4. This drug is poorly absorbed in GIT thus is given only Penicillin
parenterally for systemic infection
a. linezolid 3. A next-generation oxazolidinone, with high potency against
b. amikacin Gram-positive bacteria, including methicillin-resistant S. aureus
c. azithromycin Clindamycin
d. chloramphenicol Linezolid
Chloramphenicol
5. This inhibits topoisomerase 2 & 4 Tedizolid
a. sulfamethoxazole
b. levofloxacin 4. This protein synthesis inhibitor bind reversibly to the 30s
c. doxycycline subunit of the bacterial ribosome, blocking the binding of
d. streptogramins aminoacyl-tRNA to the acceptor site on the mRNA-ribosome
complex. It is given intravenously in a patient with
6. This bacteriostatic drug should not be given in children, Staphylococcal infection
causes teeth discoloration, and Fanconi Syndrome Clindamycin
a. amikacin Chloramphenicol
b. clarithromycin Erythromycin
c. tetracycline Tigecycline
d. clindamycin
5. This drug prevents bacterial protein synthesis by binding to
7. This drug has a high oral bioavailability, half-life of 4-6h, the 23S ribosomal RNA of the 50s subunit
reserved for multi-drug resistant organisms Chloramphenicol
a. linezolid Clindamycin
b. erythromycin Quinupritin-dalfopristin
c. clindamycin Linezolid
d. neomycin
6. This is approved for treatment of infections caused by
8. Useful in urinary tract infection but may cause kernicterus in staphylococci or by vancomycin-resistant strains of E. faecium,
infants but not E. faecalis, it is rapidly bactericidal and given
a. trimethoprim-sulfamethoxazole intravenously
b. erythromycin Linezolid
c. tetracycline Quinupristin-dalfopristin
d. gentamycin Lincosamide
Azithromycin
9. If given to infants, may cause Gray Baby Syndrome, indicated
in Salmonella infections 7. Which of the following are respiratory quinolones
a. clindamycin Moxifloxacin
b. amikacin Gemifloxacin
c. chloramphenicol Levofloxacin
d. erythromycin All choices are correct

10. A nucleic acid synthesis inhibitor which is a drug of choice 8. The following antimicrobials prevents bacterial protein
for anthrax synthesis by binding to the 50s ribosomal subunit EXCEPT
a. ciprofloxacin Macrolide
b. norfloxacin Chloramphenicol
c. spectinomycin Tetracyclines
d. levofloxacin Lincosamides

PLATING: ANTIMICROBIALS 2 9. Which of the following is a streptogramin


1. You are the doctor taking care of a patient who is taking Quinupristin-dalfopristin
levofloxacin, a fluoroquinolone antibiotic, for pneumonia. Which Linezolide
of the following choices best describes the mechanism of action Chloramphenicol
of fluoroquinolone antibiotics? Clindamycin
a. Inhibition of topoisomerase II
b. Blockage of tRNA-ribosome-mRNA complex binding 10. Patients MJ is taking this drug whose mechanism of action is
c. Disruption of mycolic acid synthesis inhibition of folic acid synthesis
d. Disruption of peptidoglycan cross-linkage Erythromycin
Trimethoprim
Levofloxacin
Cephalexin

11
19. The oral agent preferably used for preparation of the bowel
11. Ciprofloxacin belongs to what class of antibiotic prior to surgery
Fluoroquinolones Gentamycin
Aminoglycosides Neomycin
Macrolide Sisomycin
Tetracyclines Streptomycin

12. Administration of this drug is a risk factor for diarrhea and 20. The aminoglycoside popularly used before gonococcal
colitis due to C. dificile infection
Erythromycin Spectinomycin
Clindamycin Netilmicin
Tigecycline Gentamicin
Chloramphenicol Streptomycin

13. It prevents bacterial protein synthesis by binding to the 23s


ribosomal RNA of the 50s subunit and is used in methicillin-
resistant staphylococci and vancomycin resistant Enterococci
Quinupristin-dalfopristin
Azithromycin
Lincosamide
Linezolid

14. RM, 8-year-old male was seen at the OPD because of


streptococcal sore throat. Her mother claimed he had history of
allergy to amoxycillin and cephalexin. You prefer to give this
drug
Erythromycin
Clindamycin
Tigecycline
Chloramphenicol

15. A one month old infant was admitted in a hospital after


receiving a drug for 5 days. The infant had vomiting with
irregular rapid respiration, had abdominal distention and
cyanosis. The drug possibly given was
Erythromycin
Clindamycin
Tigecycline
Chloramphenicol

16. Patient A is a 15 y/o and was prescribed with ciprofloxacin,


which of the following statement is correct
Has a bioavailability of 30%
May damage growing cartilage and cause an arthropathy
Is a defluorinated analogue of nalidixic acid
Has no gram positive cover

17. Patient KL has multidrug tuberculosis, what possible


quinolone can be given to KL as second line treatment for his
tuberculosis
Moxifloxacin
Ofloxacin
Norfloxacin
Perfloxacin

18. Gray baby syndrome and irreversible aplastic anemia are


adverse effects of which drug
Chloramphenicol
Clindamycin
Linezolid
Quinupristin-dalfopristin

12
CCP: BLOOD 9. What is the vitamin that become deficient in
1. What injectable anticoagulant should be given this case?
IV to prevent additional clot formation to a. B12
prevent this case from getting worse? 10. What other vitamin deficiency can result with
a. Heparin the same symptoms?
b. Warfarin a. Folic Acid
c. Dicumarol B12 and Folic Acid
d. B and C Both needed for the maturation
2. What are the possible side effects of this drug of RBC (conversion from
(heparin)? reticulocytes to mature RBC)
a. Bleeding
b. Hair loss UNIT TEST BLOOD
c. Osteoporosis 1. The following are activators of plasminogen to
d. All of the above plasmin which will lead to fibrinolysis.
3. What is the antagonist for this drug (heparin) if a. Urokinase
side effects are noted? b. Streptokinase
a. Protamine sulfate c. Anistreplase
Case #2: Iron Deficiency Anemia d. All of the above
4. When the patient showed signs of anemia what 2. Inhibitors of clot formation
most likely is the diagnosis? a. Heparin
a. Hypochromic b. Warfarin
b. Microcytic c. Dicumarol
c. Both d. All of the above

Iron deficiency anemia Heparin


Microcytic small size RBC Has antithrombin property, facilitates
Hypochromic paler in appearance the activity of thrombin
A fibrinolytic agent
5. What is the most likely parasite that causes the Warfarin and Dicumarol
anemia? Inhibitors of vitamin K which can also
a. Hookworm (most common) lead to fibrinolysis
6. Because of chronic blood loss caused by blood 3. Heparin antagonist
sucking parasite what element is being loss by a. Protamine sulfate given if you have side
the patient leading to deficiency? effect already of heparin
a. Iron element in hemoglobin 4. Use to reverse the effects of warfarin
7. What are the different forms of this element a. Vitamin K
available for therapeutic use?
Warfarin inhibits vitamin K, so the
a. Ferrous sulfate
substance use to reverse the
b. Fumarate
effects of warfarin is vitamin K
c. Gluconate
d. All of the above 5. Microcytic hypochromic anemia
Case #3: Vitamin B12 Deficiency Anemia a. Iron deficiency
8. A patient is a known vegetarian and eventually b. B12 deficiency
showed signs of glossitis, numbness, and pallor, c. Folic acid deficiency
what is the most likely diagnosis? B12 and folic acid deficiency causes
a. Macrocytic megaloblastic anemia
b. Megaloblastic
c. Both
6. Causes platelet aggregation 4. Use to reverse the effects of warfarin
a. Adenosine diphosphate a. Vitamin K
b. Thromboxane 5. Causes platelet aggregation
c. Prostacyclin (anti-platelet aggregator) a. Aminocaproic acid
d. A and B b. Streptokinase
7. Protease inhibitor that inactivates coagulation c. Thromboxane
pathway away from the site of injury d. Urokinase
a. Antitrypsin After an injury, the blood vessel goes into
b. Macroglobulin spasm and vasoconstriction due to the
c. Both effects of Serotonin, Epinephrine, and
8. Inhibitor of fibrinolysis (procoagulants) Thromboxane released by the platelet
a. Aminocaproic acid cells
b. Tranexamic acid Platelet cell will begin to aggregate and
c. Both adhere to one another to form a Platelet
Both are procoagulants and given in patients Plug
with bleeding tendencies Enzymes involved: vWF, ADP,
Thromboxane, Phosphodiesterase,
9. Promotes activation of plasminogen to plasmin Glycoprotein IIb, Glycoprotein IIIa
a. Anistreplase
b. Streptokinase 6. Protease inhibitor that inactivates coagulation
c. Urokinase proteins away from the site of injury.
d. All of the above a. Antitrypsin
10. Dissolves the clot b. Aminocaproic acid
a. Plasmin c. Thromboxane
b. Fibrin (the clot itself) d. Glycoprotein IIb
BLOOD PLATINGS 7. Known as the Christmas factor and is affected
1. This drug does not activate plasminogen to by warfarin.
plasmin and will not lead to fibrinolysis. a. Factor IX
a. Urokinase 8. This anticoagulant accelerates the effects of
b. Streptokinase antithrombin.
c. Anestriplase a. Heparin
d. Phosphodiesterase 9. This drug inhibits activated factor Xa in the
2. Inhibitors of clot formation which indirectly common final pathway of clotting, has a rapid
block antithrombin. onset of action (half-life)
a. Heparin a.
b. Rivaroxaban
Heparin facilitates the activity of
c. Warfarin
antithrombin
d. Heparin
Indirect thrombin inhibitor
MOA to activate antithrombin to Oral direct Factor Xa Inhibitors
neutralize thrombin indirectly no Rivaroxaban
additional fibrin clot formation Apixaban
For prevention of Embolic Stroke in Atrial
3. Heparin antagonist
Fibrillation
a. Protamine sulfate Endoxaban still being develop
If you have excessive infusion of heparin Antidote Andexanet alfa
and there is already a side effect the Can block the Common Pathway of the
antagonist is protamine sulfate. Extrinsic and Intrinsic Pathways
10. A specific irreversible thrombin inhibitor from Other answers:
leech saliva. GM-CSF
a. Hirudin
11. This drug is an oral direct activated 10 inhibitor 18. The following are the hematopoietic growth
that is approved for the use in the prevention of factors except
stroke in a non-valvular atrial fibrillation. a. Epoetin Alfa
a. Apixaban b. Interleukin
12. This is the major adverse effect of heparin. c. Thrombopoietin Receptor Agonist
a. Bleeding d. Deferaserox
Heparin a fibrinolytic agent 19. All about apixaban except
Adverse effects a. It inhibits activated factor Xa
Bleeding, allergy, hair loss b. Rapid onset of action
(alopecia), osteoporosis/fractures c. Given IV
d. Prevention of stroke in non-valvular atrial
Thrombocytopenia
fibrillation
hypercoagulable state
Apixaban is taken orally
13. This drug induces release of reticulocytes from
the bone marrow. 20. The following are given parenterally except
a. Erythropoietin a. Bivalirudin
14. All but one is a megakaryocyte growth factor b. Argatroban
a. Interleukin II c. Davigatran
b. Romiplostim d. Lepirudin
c. Eltrombopag 21. The following drugs reduce platelet aggregation
d. Pegfilgastrim by inhibiting ADP pathway except
15. The introduction of this drug represents the a. Clopidogrel
milestone of the chemotherapy induced b. Ticagrelor
neutropenia. c. Aspirin
a. G-CSF d. Prasugrel
Other answers: 22. Inhibit GP IIb and IIIa except
Deferoxamine a. Tirofiban
Thrombopoietin receptor agonist b. Ticlopidine
Interleukin c. Abciximab
d. Eptifibatide
16. Approved for the secondary prevention of 23. Possible treatment for both hemophilia A and B
thrombocytopenia in patients receiving a. Factor VIII
cytotoxic therapy for treatment of non-myeloid b. Factor VIIa
cancers. c. Factor IX
a. Tetrahydrofolate d. All of the above
b. C 24. The following is a vitamin K except
c. Cyanocobalamin a. Antidote of warfarin
d. Interleukin II b. Vitamin K2 in food
17. Splenic rapture is a rare but serious c. There is a need of giving vitamin K to
complication of this myeloid growth factor PBSC newborn babies
mobilization. d. Vitamin K1 is given clinically in oral and
a. G-CSF parenteral form
b. Interleukin DYSLIPIDEMIA POST TEST CLINICAL CASE
c. Deferoxamine 1. It is classified as high intensity statin.
d. Folate a. Simvastatin
b. Atorvastatin 10 mg (should be 40-80mg)
c. Lovastatin
d. Rosuvastatin 20 40 mg HMG-CoA Reductase Inhibitor
2. Which of the ff is a modifiable risk factor? Inhibits the rate limiting step in
a. Smoking cholesterol synthesis
b. Age
c. Sex 2. All but one statin should be given at night
d. Ethnicity because cholesterol synthesis occurs
3. It is the most atherogenic lipoprotein predominantly at night.
a. LDL-c a. Rosuvastatin
4. It is the target LDL-C in patients with 3. Absorption of this statin is affected by presence
hypertension and comorbidities of food.
a. <100 g/dL a. Pravastatin
5. It is the target HDL-C to improve cardiovascular 4. The catabolism of this drug proceeds chiefly
outcomes through CYP2C9.
a. >50 a. Rosuvastatin
6. It is the most important side effect of statins 5. It functions primarily as ligands for the nuclear
that leads to emphasize in the patient transcription receptor PPAR-alpha.
a. Myopathy most common and important a. Fibrates
b. Rhabdomyolysis most common side effect 6. It inhibits VLDL secretion thus decreasing the
production of LDL in increase clearance of VLDL
You have to monitor this by monitoring serum
via the LDL pathway.
ck or one of the manifestations if tea-colored
a. Niacin
urine.
7. Which of the following is true regarding high
7. A statin that can be taken anytime density lipoprotein?
a. Rosuvastatin a. Stimulates oxidation of atherogenic
8. Which of the ff is an acute fatal complication of lipoproteins - LDL
severe hypertriglyceridemia? b. It promotes deposition of cholesterol in the
a. Hepatitis can be a complication arterial wall
b. Myositis c. Causes an impaired cholesterol retrieval
c. Rhabdomyolysis d. Helps reduce the level of atherogenic
d. Pancreatitis - most fatal lipoproteins
9. Which of the following has the highest source of 8. It is formed in the intestine and carry
triglycerides? triglycerides of dietary origin.
a. Rice a. Chylomicrons
10. It has a greater reduction in the level of 9. It is secreted by the liver and exports
triglycerides. triglycerides to peripheral tissue.
a. Fiber a. VLDL
DYSLIPIDEMIA POST TEST 10. It is an important component of metabolic
1. Which of the ff is true regarding the mechanism syndrome aside from insulin resistance,
of action of HMG-CoA reductase inhibitor? hypertension, and abdominal obesity.
a. Inhibits the last step in the steroid a. Hypertriglyceridemia
biosynthesis DYSLIPIDEMIA PLATINGS
b. Induce an increase in high affinity LDL 1. A 57-year-old male is on atorvastatin for almost
receptor 2 years you should temporarily discontinue this
c. Decreases the functional catabolic rate of drug in the event of the ff:
LDL a. Serious illness
d. Decreases hepatic extraction of LDL b. Major surgery
c. Trauma
d. All of the above
2. Gemfibrozil 600 mg once daily or BID is 8. True of homocysteine
recommended in the ff: a. Initiates proatherogenic changes in
a. In hypertriglyceridemia in which VLDL endothelium
predominates b. Can be reduced in many patients by
b. In dysbetalipoproteinemia restriction of total protein intake
c. In treating the hypertriglyceridemia that c. Reduction of high levels of homocysteine is
results from treatment with antiviral especially important in individuals with
protease inhibitors elevated levels of LP(a).
d. All choices are correct d. All of the above
3. This is the fibrate of choice for use in 9. Treatment of heterozygous
combination with statin hypercholesterinemia in children EXCEPT
a. Fenofibrate a. Maybe treated with a resin reductase
b. Gemfibrozil inhibitor
c. Clofibrate b. Treatment usually starts before 7 to 8 years
d. All choices are correct of age
4. True regarding the mechanism of action of c. Treatment starts with migration of the CNS
niacin is essentially complete
a. Inhibits VLDL secretion in turn decreasing d. The decision to treat a child is base on the
the production of LDL
b. Decreases the catabolic rate of HDL age, and other risk factors
c. Reduces fibrinogen level 10. The ff is/are true statements on diet in
d. All choices are correct managing dyslipidemia
5. A 50-year-old male with dyslipidemia was a. Cholesterol and saturated trans fats are the
prescribed a drug for elevated cholesterol he principal factors that increase LDL
developed constipation and bloating after 15 b. Total fat sucrose especially fructose
mL granules/day for a week, the drug taken is increases VLDL
possibly c. Alcohol can cause significant
a. Colestipol hypertriglyceridemia by increasing hepatic
b. Fenofibrate secretion of VLDL
c. Ezetimibe d. All of the above
d. All choices are correct 11. Secondary causes of hypertriglyceridemia
Constipation and bloating the most common a. Alcohol ingestion
adverse effect of bile acid sequestrant b. Hypothyroidism
c. Corticosteroid excess
6. This is added to statin therapy providing d. All of the above
additional effect decreasing LDL levels and 12. The most effective agent for increasing HDL and
further reducing the dimensions of the only agent that may reduce lipoprotein A
atherosclerotic plaques. a. Niacin
a. Colestipol b. Atorvastatin
b. Fenofibrate c. Gemfibrozil
c. Ezetimibe d. Probucol
d. All of the choices are correct 13. It denotes increased levels of triglycerides
7. Incorrect statement of LDL receptor deficient a. Hyperlipidemias
familial hypercholesterinemia b. Hyperlipoproteinemia
a. This is an autosomal dominant trait c. Hyperlipemia
b. Triglycerides are usually abnormal too d. Dyslipidemia
c. Tendon xanthomas are often present 14. It is the major clinical sequalae of
d. Coronary disease tends to occur hyperlipidemias
prematurely a. Cholelithiasis
b. Acute pancreatitis 21. This combination is sometimes useful in treating
c. Bleeding patients with familial combined hyperlipidemia
d. Diabetes who are intolerant of niacin and statins but may
15. It is a cellular component in atheromatous increase the risk of cholelithiasis
plaque which are transformed macrophages a. Fibric acid derivatives + bile acid binding
smooth muscle with cholesterol esters resins
a. Foam cells b. HMG-CoA reductase inhibitor + bile acid
b. LDL resin
c. Triglycerides c. Reductase inhibitor + fibrates
d. VLDL d. Niacin + reductase inhibitor
16. It participates in the retrieval of cholesterol 22. Which of the following drugs is not part of the
from the artery wall and inhibit oxidation of combination treatment which acts in the
atherogenic lipoprotein complementary fashion to normalize
a. LDL cholesterol in patients with severe disorders
b. VLDL involving elevated LDL?
c. Triglycerides a. Fibrates
d. HDL b. Resins
17. It is secreted by the level and transports c. Acetamide
triglycerides to peripheral tissue d. Niacin reductase inhibitor
a. Chylomicrons 23. The PCSK9 humanized monoclonal antibodies
b. VLDL which inhibits catabolism of LDL receptor
c. LDL a. Evolocumab
d. HDL b. Acetamide
18. An MTP inhibitor but is currently restricted to c. Lomitapide
patients with homozygous familial d. Mipomersen
hypercholesterolemia ANTIMICROBIALS CCP
a. Lomitapide 1. Available cephalosporin for surgical prophylaxis
b. Mipomersen of acute appendicitis
c. Alirocumab a. Cefaclor
d. Torcetrapib b. Loracarbef
19. Which of the following statements is correct c. Cefmetazole
about drug combination in the treatment of d. Cefazolin
dyslipidemia? 2. Non-pharmacologic management of cellulitis
a. When VLDL levels are significantly increased EXCEPT
during the treatment of a. Properly cover the skin lesion
hypercholesterolemia with resins b. Keeping the area clean
b. When VLDL level is elevated and VLDL is c. Avoid scratching the lesion
decreased initially d. Apply hydrocortisone cream to lesion
c. When LDL or VLDL level are normalized with
Hydrocortisone is a steroid so; it can
a single agent
decrease the effect of your
d. When levels of LP and HDL are both
antibiotics you cannot apply
elevated
hydrocortisone on the lesion
20. An antisense oligonucleotide that targets APO
B100 mainly in the liver 3. Component of CURB criteria EXCEPT
a. Mipomersen a. Temperature
b. Lomitapide b. Blood pressure
c. Alirocumab c. Respiratory rate
d. Torcetrapib d. Urea
4. The following agent is indicated for patients 1. Correct statement about cefazolin
with acute appendicitis EXCEPT a. The only 1st generation parenteral
a. Cefazolin cephalosporin
b. Cefuroxime b. A good antibiotic for meningitis - wrong bc
c. Combination of amoxicillin and clavulanic it cannot penetrate to the CNS freely
acid c. Penetrates freely to the CNS
d. Fluconazole for fungal infection d. Belongs to 2nd generation cephalosporin
5. Mechanism of beta lactamase inhibitor (beta 2. Which of the given choices is not correctly
lactamase inhibitor is classified as cell wall matched?
inhibitor) a. Carbapenems telavancin
a. Interferes with bacterial cell wall synthesis b. Lipopeptides daptomycin
b. Inhibit protein synthesis by binding with 50S c. Cephalosporin cefotetan
ribosome d. Glycopeptides vancomycin
c. Inhibit protein synthesis by binding to 30S 3. A group of cephalosporin with good body fluid
ribosome subunit and tissue penetration
d. Inhibit folic acid synthesis a. 1st generation
6. Cefuroxime belongs to what generation of b. 2nd generation
cephalosporins c. 3rd generation
a. 1st generation d. 4th generation
b. 2nd generation 4. A 2nd generation cephalosporine which can be
c. 3rd generation combined with macrolides for the treatment of
d. 4th generation community acquired pneumonia
7. Oral antibiotic indicted for case 3 with mild a. Loracarbef
localized staphylococcus infection b. Ceftazidime
a. Cloxacillin c. Cefalexin
b. Procaine penicillin d. Cefuroxime
c. Benzathine penicillin 5. Excessive level of this carbapenems may lead to
d. Ceftazidime seizure in patients with renal failure
a. Ertapenem
Procaine penicillin, benzathine penicillin, b. Imipenem
ceftazidime are all susceptible to beta c. Meropenem
lactamase enzyme d. Doripenem
Cloxacillin beta lactamase resistant 6. All but one is true about red man syndrome
a. It is one of the adverse effects of aztreonam
8. All but one is common adverse effects of b. Due to the release of histamine
penicillin and cephalosporin c. It is an infusion related flushing
a. Allergy (most common) d. Can be treated with anti-histamine
b. Nausea 7. The following choices are 2 nd generation
c. Abdominal discomfort cephalosporine EXCEPT
d. Cellulitis these are used to treat cellulitis a. Cefoxitin
9. Penicillin is not indicated in the following b. Cefuroxime
conditions c. Cefotetan
a. Pneumonia d. Cephalexin 1st generation
b. Meningitis 8. A glycopeptide for the treatment of C. difficile
c. Conjunctivitis infection
d. Benign prostatic hypertrophy a. Vancomycin
10. C in CURB risk assessment of pneumonia means b. Telavancin
a. Confusion c. Teicoplanin
ANTIMICROBIALS 1 POST TEST d. Oritavancin
9. Penicillin is poorly absorbed in the following b. Dicloxacillin
organ EXCEPT c. Ampicillin
a. Skin d. Amoxicillin
b. CNS 7. Prevention at peptidoglycan elongation and
c. Eyes crosslinking resulting to inhibition of cell wall
d. Prostate synthesis is a mechanism of action of all of the
10. This oral form of penicillin is indicated only in following EXCEPT
minor infection because of poor bioavailability a. Vancomycin
a. Procaine penicillin G b. Telavancin
b. Benzathine penicillin G c. Teicoplanin
c. Penicillin V d. Carbapenems (a beta lactam drug)
d. Nafcillin 8. Telavancin and teicoplanin are alike in that
ANTIMICROBIALS 1 PLATING both
1. Structural integrity of this nucleus is essential a. Inhibit call wall synthesis by binding to the
for biologic activity of penicillin d-Ala-d-Ala terminus of peptidoglycan in the
a. Penicilloic acid growing cell wall (cause prevention of
b. 6-amino penicillanic acid peptidoglycan elongation)
c. Beta lactamase b. Commonly used for the treatment of
d. Secondary amino group hospital acquired pneumonia
2. All but one is true statement of a penicillin c. Commonly cause infusion related flushing
a. Blood level of penicillin decreases with due to the release of histamine (red man
simultaneous administration of probenecid syndrome vancomycin)
b. Oral penicillin should be given 1-2 hours d. Disrupt the bacterial cell wall membrane
before or after a meal and increase permeability
c. Should viral be used for viral infection 9. Which of the following ____ beta lactamase
d. Is available in oral and parenteral form inhibitor
3. A single intramuscular injection of this penicillin a. Sulbactam (and clavulanic acid are beta
is an effective treatment of beta hemolytic lactamase inhibitors)
streptococcal pharyngitis every 3-4 weeks in b. Ertapenem
order to prevent the complication c. Oritavancin
a. Procaine d. Teicoplanin
b. Ticarcillin 10. Nausea, vomiting, and diarrhea are common
c. Piperacillin adverse effects or what?
d. Benzathine a. Vancomycin
4. The first-generation cephalosporin b. Carbapenem
a. Cephalothin c. Sulbactam
b. Cefuroxime (2nd generation) d. Telavancin
c. Cefoxitin (2nd generation) 11. Seizures observed in carbapenems is most likely
d. Ceftazidime (3rd generation) due to what?
5. Resistance to penicillin and other beta lactam is a. Doripenem
true to all but one b. Ertapenem
a. Inactivation of antibiotic by beta lactamase c. Meropenem
b. Alteration of target PBP d. Imipenem
c. Good penetration of drug to target PBP 12. Vancomycin is usually given parenterally
d. Antibiotic efflux because it is poorly absorbed in the GIT, it is
6. Gastro intestinal absorption of this penicillin is only given orally for the treatment of what?
erratic therefore it is not suitable for oral a. Colitis (cause by Clostridium difficile)
administration b. Endocarditis (cause by Methicillin
a. Nafcillin susceptible strains)
c. Meningitis (cause by penicillin resistance 18. MR, 30, female, is scheduled for surgery and is
stains) given an intravenous infusion of 1g of this 1st
d. Necrotizing pneumonia (cause by generation parenteral cephalosporin
Staphylococcus aureus) a. Cefazolin (used for surgical prophylaxis)
13. This is the mechanism of action of daptomycin b. Cefalexin
a. It binds to cell membrane causing c. Cefoxitin
depolarization and rapid cell death d. Cefaclor
b. Prevents bacterial cell wall synthesis by 19. A second-generation cephalosporin
binding to and inhibiting cell wall a. Cefaclor
transpeptidases b. Cephalexin (1st generation)
c. Inhibits the cytoplasmic enzyme phenol c. All are correct
pyruvate transferase d. Cefixime (3rd generation)
d. Inhibit cell wall synthesis by binding to d- 20. A third-generation cephalosporine
Ala-d-Ala terminus of nascent peptidoglycan a. Ceftriaxone
(mechanisms of action of telavancin and b. Cefuroxime
teicoplanin) c. Cefaclor
14. This is the mechanism of action of Fosfomycin d. Cefalexin
a. Binds to cell membrane, causing 21. RP, 28-year-old female, was diagnosed with
depolarization and rapid cell death pelvic inflammatory disease, she will benefit
b. Inhibits cell wall synthesis by binding to the from this 2nd generation cephalosporin used to
D-Ala-D-Ala terminus of nascent treat anaerobic infection
peptidoglycan a. Cefoxitin (cefotetan, cefmetazole)
c. Prevents bacterial cell wall synthesis by b. Cefuroxime
binding to and inhibiting cell wall c. Cefotaxime
transpeptidase d. Ceftazidime
d. Inhibits the cytoplasmic enzyme 22. The only 3rd generation cephalosporine with
enolpyruvate transferase useful activity against P. aeruginosa
15. This drug is usually use in the treatment of a. Ceftazidime
tuberculosis cause by stains of Mycobacterium b. Cefixime
tuberculosis that are resistant to this agent c. Cefalexin
a. Cycloserine d. Cefepime (4th generation Cefpirome)
b. Daptomycin 23. Adverse effect of cephalosporine
c. Fosfomycin a. Anaphylaxis
d. Bacitracin b. Hemolytic anemia
16. This antibiotic is highly nephrotoxic and is only c. Granulocytopenia
used as a topical agent d. All are correct
a. Cycloserine ANTIMICROBIALS 2 POST TEST
b. Daptomycin 1. This drug blocks the 30S in protein synthesis
c. Fosfomycin a. Gentamycin
d. Bacitracin b. Clindamycin 50S
17. This drug is not used in the management of c. Chloramphenicol 50S
pneumonia due to its activation of pulmonary d. Clarithromycin 50S
surfactant 2. This 50S inhibitor is a microlides
a. Daptomycin a. Erythromycin
b. Vancomycin b. Troleandomycin 30S
c. Meropenem c. Gentamycin 30S
d. Fosfomycin d. Netilmicin 30S
3. These drugs are auto toxic and nephrotoxic
a. Aminoglycosides
b. Tetracycline (photophobia/GIT symptoms) d. Norfloxacin (not indicated for systemic
c. Chloramphenicol (bone marrow depression, infection)
grey baby syndrome) ANTIMICROBIALS 2 PLATING
d. Trimethoprim sulfamethoxazole (GIT 1. This drug prevents bacterial protein synthesis
symptoms) by binding to the 23S ribosomal subunit of 50S
4. This drug is poorly absorbed in the GIT and does subunit
given only parenterally for systemic infection a. Linezolid
a. Linozelid b. Quinupristin-Dalfospristin
b. Amikacin c. Chloramphenicol
c. Chloramphenicol d. Clindamycin
d. Azithromycin 2. Which of the following is streptogramin?
5. This inhibits topoisomerase II and IV a. Quinupristin-Dalfospristin
a. Levofloxacin (nucleic acid inhibitor) b.
b. Sulfamethoxazole (inhibits dihydropteroate 3. You are the doctor taking care of a patient who
synthase) is taking levofloxacin, a fluoroquinolone
c. Doxycycline antibiotic, for pneumonia. Which of the
d. Streptogramin following choices best describes the mechanism
6. This bacteriostatic drug should not be given in of action of fluoroquinolone antibiotics?
children, causes teeth discoloration and Fanconi a. Inhibition of topoisomerase II
Syndrome b. Distribution of mycolic acid synthesis
a. Tetracycline c. Distribution of peptidoglycan cross-linkage
b. Clarithromycin d. Blockage of tRNA-ribosome-mRNA complex
c. Clindamycin binding
d. Amikacin 4.
7. This drug has a high oral bioavailability, half life 5.
of 4-6 h, reserved for multi drug resistant 6.
organisms 7. Gray baby syndrome and irreversible aplastic
a. Linezolid anemia are adverse effects of which drug?
b. Clindamycin a. Chroramphenicol
c. Neomycin b. Quinupristin-dalfopristin
d. Erythromycin c. Linezolid
8. Useful in urinary tract infection but may cause d. Clindamycin
kernicterus in infants 8. This protein synthesis inhibitor binds reversibly
a. Tetracycline to the 30S subunit of the bacterial ribosome,
b. Gentamycin blocking the binding of aminoacyl-tRNA to the
c. Erythromycin acceptor site on the mRNA-ribosome
d. Trimethoprim sulfamethoxazole complexes. It is given intravenously in a patient
9. If given to infants may cause Gray Baby with Staphylococcal infection.
Syndrome, indicated in Salmonella infections a. Tigecycline
a. Clindamycin (Pseudomonas colitis) b. Erythromycin
b. Amikacin (autotoxicity & nephrotoxicity) c. Clindamycin
c. Chloramphenicol d. Chloramphenicol
d. Erythromycin 9. RM, 8-year-old male was seen at the OPD
10. A nucleic acid synthesis inhibitor which is a drug because of streptococcal sore throat. Her
of choice for anthrax mother claimed he had history of allergy to
a. Levofloxacin amoxycillin and a cephalexin. You prefer to give
b. Ciprofloxacin this drug
c. Spectinomycin a. Erythromycin
b. Tigecycline
c. Clindamycin a. Tigecycline
d. Chloramphenicol b. Erythromycin
10. Patient MJ is taking this drug whose mechanism c. Clindamycin
of action is inhibition of folic acid synthesis d. Chloramphenicol
a. Trimethoprim 17. Ciprofloxacin belongs to what class of
b. Cephalexin antibiotic?
c. Levofloxacin a. Aminoglycosides
d. Erythromycin b. Fluoroquinolones
11. Patient KL has multidrug tuberculosis, what c. Macrolides
possible quinolone can be given to KL as d. Tetracyclines
second-line treatment for his tuberculosis? 18. The oral agent preferably used for preparation
a. Norfloxacin of the bowel prior to surgery
b. Pefloxacin a. Sisomycin
c. Ofloxacin b. Neomycin
d. Moxifloxacin c. Gentamycin
12. Patient A is 15 years old and was prescribed d. Streptomycin
with ciprofloxacin, which of the following 19. Which of the following are respiratory
statement is correct? quinolones?
a. Is a defluorinated analogue of nalidixic acid a. Levofloxacin
b. May damage growing cartilage and cause an b. All choices are correct
arthropathy c. Gemifloxacin
c. Has a bioavailability of 30% d. Moxifloxacin
d. Has no gram-positive cover 20. Minimally absorbed in the GIT this is preferred
13. Administration of this drug is a risk factor for to be given parenterally for systemic infection
diarrhea and colitis due to C. difficile a. Aminoglycosides
a. Chloramphenicol b. Erythromycin
b. Clindamycin c. Cephalosporins
c. Erythromycin d. Penicillin
d. Tigecycline ANTIVIRAL/ANTIFUNGAL POST TEST
14. An aminoglycoside that is considered as one of 1. The major extracellular site of action of antiviral
the effective anti TB drugs but is given only IM a. Viral attachment and entry (viral release)
a. Streptomycin b. Upon protein synthesis, packaging, and
b. Isoniazid assembly (intracellular)
c. Rifampicin c. Integration and transcription (intracellular)
d. Ethambutol 2. Mechanism of action of protease inhibitors
15. This is approved for treatment of infections a. Binds to host CCRH protein which facilitates
caused by staphylococci or by vancomycin- entry of the virus inside the cell
resistant strains of E. faecium, but not E. b. It inhibits the post-translational cleavage of
faecalis. It is rapidly bactericidal and is given GAG-POL polyprotein resulting to immature
intravenously non-infectious virus
a. Quinupristin-dalfopristin c. Binds to HIV 1 & 2 integrase enzyme
b. Linezolid needed for the integration of the virus to
c. Lincosamide the host
d. Azithromycin d. Binds directly to HIV 1 reverse transcriptase
16. A one-month-old infant was admitted in a resulting to RNA & DNA dependent
hospital after receiving a drug for 5 days. The polymerase (function of NNRTI)
infant had vomiting with irregular rapid e. Competitive inhibition of HIV reverse
respiration, had abdominal distention and transcriptase (function of NRTI)
cyanosis. The drug possibly given was: 3. Mechanism of action of NNRTI
a. Binds to HIV 1 & 2 integrase enzyme e. Isavuconazole
needed for the integration of the virus to 10. Mechanism of action is blockade of B Glucan
the host Synthesis
b. Inhibit post translational cleavage of gag-pol a. Ketoconazole
polyprotein resulting to immature non b. Flucytosine
infectious virus c. Caspofungin
c. Bind directly to HIV 1 Reverse Transcriptase d. Nystatin
resulting to RNA & DNA dependent DNA e. Terbinafine
polymerase ANTIVIRAL/ANTIFUNGAL CCP
d. Competitive inhibition of HIV 1 reverse 1. Clinical presentation of case 1 with
transcriptase Oropharyngeal Candidiasis
4. Entry inhibitor a. Bleeding ulcerated lesions on the tongue
a. Efavirenz NNRTI and oropharynx
b. Maraviroc b. White patches covering the buccal mucosa,
c. Ritonavir PI tongue and posterior pharyngeal wall
d. Zidovudine NRTI c. Elevated hyperemic patches on the tongue,
e. Abacavir NRTI buccal mucosa and oropharynx
5. Used in the treatment of Hepatitis B infection d. Erythematous buccal mucosa and
a. Dasabuvir oropharyngeal wall
b. Entecavir e. Yellowish mucopurulent discharge on the
c. Daclatasvir post pharyngeal wall
d. Ombitasvir 2. Topical drug of choice for Oropharyngeal
e. Ledipasvir Candidiasis
6. Primarily used for Cytomegalovirus (CMV) a. Griseofulvin
retinitis b. Nystatin
a. Valacyclovir c. Ketoconazole
b. Acyclovir d. Micafungin
c. Famciclovir e. Amphotericin B
d. Ganciclovir 3. MOA of the topical drug of choice for
e. Pencyclovir Oropharyngeal Candidiasis (Nystatin)
7. Neuraminidase inhibitor used for both Influenza a. Blocks B Glucan synthase
A&B b. Blocks fungal CYP 450
a. Peramivir c. Inhibit squalene epoxidation
b. Ribavirin d. Interfere with fungal DNA and RNA
c. Dasabuvir synthesis
d. Elbasvir e. Forms pores in cell wall membrane
e. Ledipasvir 4. Alternative topical drug for Oropharyngeal
8. Azole of choice for Aspergillus infection Candidiasis
a. Fluconazole a. Amphotericin B
b. Posaconazole b. Micafungin
c. Isavuconazole c. Griseofulvin
d. Itraconazole d. Clotrimazole
e. Voriconazole e. Caspofungin
9. Broadest spectrum Azole and the first azole 5. Diagnosis for the case with Erythematous
with significant activity vs. Mucormycosis maculopapular rashes on the thoracic with
a. Voriconazole occasional fluid filled blisters accompanied by
b. Itraconazole moderate to severe pain
c. Posaconazole a. Shingles
d. Fluconazole b. Hepatitis C
c. CMV
d. HIV
e. Measles
6. True statement regarding the case with
Erythematous maculopapular rashes on the
thoracic area with occasional fluid filled blisters
accompanied by moderate to severe pain
a. Antiviral creams are generally not
recommended
b. Oral antibiotics are given as prophylaxis to
prevent secondary bacterial infection
c. Cytomegalovirus is the causative agent
d. Oral Abacavir is the drug of choice
e. Trifluridine creams are applied to the
lesions within 12 hours to shorten healing
time
7. Used in the treatment of Herpes Zoster or
Varicella Zoster infection
a. Indinavir
b. Abacavir
c. Atazanavir
d. Acyclovir (DOC)
e. Tenofovir
8. Causative organism in the case with edematous
external auditory canal and fuzzy black cotton
like material
a. Cryptococcus
b. Aspergillus
c. Candida
d. Clostridium
e. Blastomyces
9. Drug of choice for Otomycosis (case 2)
a. Amphotericin B
b. Griseofulvin
c. Clotrimazole
d. Ketoconazole
e. Terbinafine
10. MOA of the drug of choice for Otomycosis (Case
2)
a. Interfere with fungal DNA and RNA
synthesis
b. Blocks B Glucan synthase
c. Reduce ergosterol synthesis by inhibiting
fungal CYP 450
d. Inhibit squalene epoxidation
e. Forms pores in fungal cell wall
ANTI-TB ANTI-PARASITIC DRUGS POST TEST Answer: C./ E. Histolytica, a protozoan; A & B are bacteria
1. This remains as the primarily and widely used / D, Giardia also protozoan, can also cause diarrhea
diagnostic tool for PTB and available in most health
6. This drug is indicated in luminal and extra intestinal
centers in the country.
amebiasis:
A. CBC/ APC
A. metronidazole
B. Chest xray
B. mebendazole
C. DSSM
C. chloroquine
D. CT Scan
D. diloxanide furoate
Answer is C. DSSM (Direct Sputum Smear microscopy)
Answer: A, metronidazole / Mebendazole for round
2. This antiTB drug is bactericidal and avidly binds with worm inf/ chloroquine for extra intestinal amebiais and
the DNA dependent RNA polymerase: diloxanide furoate is a luminal amecidal drug, used in
A. Isoniazid asymptomatic passers of cyst
B. Rifampin
7. This drug causes headache, paresthesia and also
C. Pyrazinamide
causes disulfiram like effect:
D. Ethambutol
A. metronidazole
ANSWER IS: B, Rifampin / INH: inhibitics mycolic acid B. mebendazole
synthesis/ Pyrazinamide: inhibits cell membrane/ C. chloroquine
Ethambutol: inhibits arabinosyl galactan D. diloxanide furoate

3. This drug should not be given in young children


Answer: A metronidazole/ mebendazole, mild, GIT
because it can cause optic neuritis:
symptoms, same as diloxnide furoate/ Chloroquine can
A. Isoniazid
cause hemolysis
B. Rifampin
C. Pyrazinamide 8. The following is considered drug of choice for patient
D. Ethambutol with ascaris lumbricoides infection:
A. mebendazole
Answer: D ethambutol/ INH peripheral neuritis,
B. albendazole
jaundice/Pyrazinamide, hepatotoxi, hyperuricemia/
C.pyrantel pamoate
Ethambutol: optic neuritis
D. any of the choices is correct
4. This drug should not be given together with
phenytoin because it may cause diplopia and ataxia: Answer: D all of this can be used as the drug of choice for
A. Isoniazid ascariasis
B. Rifampin
9. This drug act by inhibition of microtubule assembly
C. Pyrazinamid
A. mebendazole
D. Ethambutol
B. albendazole
Answer: A, Isoniazid. Pharmacokinetics drug interaction C.pyrantel pamoate
by metabolism, INH inhibits metabolism of phenytoin D. metronidazole
causing cumulative toxicity to cause diplopia and ataxia.
Rifampin is an inducer of hepatic microsomal enzymes Answer is B. Albendazole/ mebendazole acts by
and can shorten the half-life of many drug. selectively inhibiting microtubule synthesis and glucose
uptake in nematodes/ pyrantel pamoate: stimulates
5. Intestinal amebiasis is caused by: nicotinic receptorspresent at neuromuscular junctions of
A. shigella sp nematodes. Metronidazole diffuses into the organism,
B. salmonella sp inhibits protein synthesis by interacting with DNA and
C. Entamoeba histolytic causing a loss of helical DNA structure and strand
D. Giardia lambdia breakage.
10. This is a primary drug for ascariasis, hookworm, 4. The drug of choice for flukes and tapeworms:
pinworm, and whipworm infections and an alternative A. Ivermectin
drug for treatment of threadworm infections, filariasis, B. Albendazole
and both visceral and cutaneous larva migrans C. Diethylcarbamazine
A. mebendazole D. Praziquantel
B. albendazole
C.pyrantel pamoate ANSWER IS D: PLEASE SEE DISCUSSION NO.6
D. metronidazole
5. The following are considered drug of choice for
Ascaris lumbricoides, EXCEPT:
Answer: B albendazole/ Mebendazole is a primary drug
A. Albendazole
for treatment of ascariasis and for pinworm and
B. Mebendazole
whipworm infections/ Pyrantel Pamoate is a drug of
C. Pyrantel pamoate
choice for hookworm and roundworm infections and an
D. Ivermectin
alternative drug for pinworms It is a drug of choice for
hookworm and roundworm infections and an alternative
ANSWER IS D: PLEASE SEE DISCUSSION IN NO. 6.
drug for pinworms/Metronidazole is a treatment for
amebiasis. 6. The drug of choice for strongyloidiasis and
onchocercosis:
1. Symptomatic mild to moderate amoebiasis is best
A. Ivermectin
managed with:
B. Albendazole
A. Giving of metronidazole
C. Diethylcarbamazine
B. Giving of diloxanide furoate
D. Praziquantel
C. Giving of both metronidazole and diloxanide furoate
D. Giving of primaquine
The answer is A
Ivermectin is the drug of choice for onchocerciasis,
FOR THE
cutaneous larva migrans, strongyloidiasis, and some
RECOMMENDATIONS OF DOC.
forms of filariasis. Diethylcarbamazine is the drug of
2. Drug of choice for the treatment of pneumocystis choice for several filarial infections including those
jirovecii pneumonia: caused by Wucheria bancrofti and Brugia malayi and for
A. Rifabutin eye worm disease (Loa loa). Praziquantel has a wide
B. Trimetoprim-sulfamethoxazole anthelminthic spectrum that includes activity in both
C. Chloroquine trematode and cestode infections. It is the drug of choice
D. Azithromycin in schistosomiasis (all species), clonorchiasis, and
paragonimiasis and for infections caused by small and
ANSWER IS B. TMP SMZ/ FOR RIFABUTIN a replacement large intestinal flukes. The drug is active against
of rifampin in HIV patient (no drug interaction with immature and adult schistosomal forms. Praziquantel is
antiviral drugs) Chloroquine antimalarial same with also 1 of 2 drugs of choice (with niclosamide) for
azithromycin drug infections caused by cestodes (all common tapeworms)
and an alternative agent (to albendazole) in the
3. Malarial relapse is due to the presence of
treatment of cysticercosis/is a primary drug for
hypnozoites or tissue schizonts, often seen with P. vivax
ascariasis, hookworm, pinworm,and whipworm
and ovale and this can be prevented by giving:
infections and an alternative drug for treatment of
A. Quinine
threadworm infections, filariasis, and both visceral and
B. Chloroquine
cutaneous larva migrans
C. Primaquine
D. Mefloquine
Answer C. Primaquine is indicated in relapsing type of
malaria is a tissue schizonticidal drug. Given after the
dose of chloroquine, or mefloquine or quine, which are
preventive or treatment of active malarial cases.
7. A 10-year-old boy has uncomplicated pulmonary 9. Once-weekly administration of which of the
tuberculosis. After initial hospitalization, he is now following antibiotics has prophylactic activity against
being treated at home with isoniazid, rifampin, and bacteremia caused by M avium complex in AIDS
ethambutol. Which statement about this case is patients?
accurate? A. Acedapsone
B. Azithromycin
A. A baseline test of auditory function test is essential
C. Clarithromycin
before drug treatment is initiated
D. Kanamycin
B. His mother, who takes care of him, does not need
INH prophylaxis
The answer is B. Because of its long elimination half-life
C. His 3-year-old sibling should receive INH
(3 4 d), weekly administrationof azithromycin has
prophylaxis
proved to be equivalent to daily administration of
D. Polyarthralgia is a potential adverse effect of the
clarithromycin when used for prophylaxis against M
drugs the boy is taking
avium complex in AIDS patients. Acedapsone is a
The answer is C. repository form of dapsone used in leprosy.

A baseline test of ocular (not auditory) function may be 10. This drug is NOT included in the first line of
useful before starting ethambutol. None of the drugs treatment of Pulmonary Tuberculosis virgin cases:
prescribed is associated with nephrotoxicity. A. Isoniazid
Polyarthralgia is a common adverse effect of B. Amikacin
pyrazinamide that was not prescribed in this case. C. Rifampin
Periodic tests of liver function may be advisable in D. Ethambutol
younger patients who are treated with INH plus rifampin,
especially if higher doses of these drugs are used. Answer is B which is a second line of anti-TB drug.
Prophylaxis with INH is advisable for all household
ANTI-TB ANTI-PARASITIC DRUGS PLATINGS
members and very close contacts of patients with active
1. This is the drug of choice in the eradication of
tuberculosis, especially young children.
dormant liver form of P. vivax and P. ovale
8. Which statement about antitubercular drugs is A. Primaquine
accurate? B. Chloroquine
C. Quinine
A. Antimycobacterial actions of streptomycin involve D. Piperaquine
inhibition of arabinosyltransferases
B. Cross-resistance of M tuberculosis to isoniazid and 2. This is now widely used as the standard
pyrazinamide is common treatment for uncomplicated falciparum malaria
C. Ocular toxicity of ethambutol is prevented by in most endemic areas.
thiamine A. Artemisinin-based combination therapy
D. Resistance to ethambutol involves mutations in B. Chloroquine
the emb gene C. Quinidine
The answer is D D. Mefloquine
Arabinosyltransferase is inhibited by ethambutol (not
streptomycin) and resistance involves alterations in the emb 3. This drug is commonly used as a
gene. Ocular adverse effects of ethambutol are dose- chemoprophylactic agent for malaria in areas
dependent and usually reversible when the drug is with chloroquine-resistant strains.
discontinued. Thiamine is not protective. There is minimal A. Mefloquine
cross-resistance between pyrazinamide and other B. Quinidine
antimycobacterial drugs. Pyrazinamide uniformly causes
C. Atovaquone
hyperuricemia, but this is not a reason to halt therapy even
D. Piperaquine
though the drug may provoke gouty arthritis in susceptible
persons.
4. This is the treatment of choice for Trichomonas 11. What aminoglycoside previously belonging to 1st
vaginalis. line anti-TB drugs, but re-classified by Katzung
A. Metronidazole 14th edition is used against atypical
B. Nifurtimox mycobacterium infection?
C. Trimethoprim-sulfamethoxazole A. Streptomycin
D. Paromomycin B. Kanamycin
C. Tobramycin
5. Treatment of choice for visceral leishmaniasis D. Neomycin
A. Sodium stibogluconate
B. Tinidazole 12. Bacteriostatic agent: (P.910)
C. Iodoquinol A. Clindamycin
D. Trimethoprim-sulfamethoxazole B. Metronidazole
C. Isoniazid
6. Treatment of choice for Giardia lamblia D. Quinolones
A. Metronidazole
B. Sodium stibogluconate 13. Antimicrobial agent that is contraindicated in
C. Nifurtimox patients with renal impairment (P.911)
D. Trimethoprim-sulfamethoxazole A. Nitrofurantoin
B. Acyclovir
7. Atypical mycobacterium kansasii is resistant to C. Erythromycin
A. Pyrazinamide D. Metronidazole
B. Rifampicin
C. Ethambutol 14. Adverse reactions to antimicrobial agents occur
D. Isoniazid with increased frequency in this group (P.912)
A. Neonates
8. What antibiotic can be used against B. Geriatric patients
Mycobacterium Avium which is a common cause C. Renal failure patients
of disseminated disease in late stage of AIDS D. All are correct
A. Azithromycin
B. Penicillin 15. Rationale for combination antimicrobial therapy
C. Ciprofloxacin (P.912)
D. Cefuroxime A. To provide broad-spectrum empiric
therapy in seriously ill patients
9. What sulfonamide is used to treat B. To decrease the emergence of resistant
Mycobacterium Leprae causing leprosy strains
A. Dapsone C. To treat polymicrobial infections
B. Trimoxazole D. All are correct
C. Sulfanilamide
D. Sulfadiazine 16. Common pathogen in nonperforated
appendectomy (P.914)
10. What first line anti-TB drug is used for leprosy A. Enteric gram-negative rods
A. Rifampicin B. Aerobes
B. Isoniazid C. Staphylococci
C. Ethambutol D. Streptococci
D. Pyrazinamide
17. Excellent nonsurgical antimicrobial prophylaxis 21. A 27-year-old male was diagnosed to have an
and drug of choice for close contacts of a patient active case of pulmonary tuberculosis. He was
with meningococcal infection started on anti-TB treatment. He returned to the
A. Ciprofloxacin OPD the following day, complaining that his
B. Amoxicillin urine was red orange in color. This drug could
C. Rifampin have caused this effect:
D. Tetracycline A. Isoniazid
(P.842-848) B. Rifampin
18. This is NOT included in the recommendation for C. Ethambutol
traditional first-line agents for treatment of D. Pyrazinamide
tuberculosis: E. Streptomycin
A. Isoniazid
B. Ethambutol 22. a 60-year-old male has been on anti-TB drugs for
C. Rifampin almost 3 weeks when he developed loss of
D. Pyrazinamide appetite, nausea, vomiting, jaundice, and right
E. Streptomycin upper quadrant pain. The attending physician
considered the symptoms as adverse effects of
19. This 2nd line of anti-TB drug is given at the drug he is taking. This could be due to:
300mg/day should be reduced to 150mg/d if A. isoniazid
used concurrently with a protease inhibitor or B. rifampin
cobicistat; and 600mg/d with efavirenz C. ethambutol
A. Amikacin D. pyrazinamide
B. Cycloserine E. streptomycin
C. Ethionamide
D. Para-amino salicylic acid 23. Aside from tuberculosis, rifampin can be used in
E. Rifabutin the following conditions, EXCEPT:
A. Use in bacterial infections
20. This is NOT the correct mechanism of action of B. To eliminate meningococcal carriage
the drug: C. Prophylaxis in contacts of children with
A. Isoniazid: blocks mycolic acid synthesis Haemophilus influenza type B disease
B. Rifampin: bind to the alpha subunit of D. In combination therapy is also used for
bacterial DNA-dependent RNA treatment of serious staphylococcal
polymerase and thereby inhibits RNA infections such as osteomyelitis,
synthesis prosthetic joint infections, and
C. Ethambutol: inhibits mycobacterial prosthetic valve endocarditis
arabinosyl transferases E. Use in fungal infections
D. Pyrazinamide: disrupts mycobacterial
cell membrane metabolism and
transport functions
E. Streptomycin: inhibits topoisomerase 2
&4
CLINICAL USES OF ANTIMICROBIALS/ 4. An application of an agent to living tissue for the
MISC. ANTIMICROBIALS purpose of preventing infection:
1. A nitro imidazole antiprotozoal drug A. Antisepsis
A. Metronidazole B. Decontamination
B. Fidaxomicin C. Disinfection
C. Rifaximin D. Pasteurization
D. Mupirocin
- Decontamination: destruction or marked
Fidaxomicin: narrow spectrum, macrocyclic antibiotic reduction in the number or activity of
Rifaximin: derivative of rifampin microorganisms
Mupirocin: pseudominic acid - Disinfection: chemical or physical treatment
that destroys most vegetative microbes or
1. Oral agent that exert antibacterial activity in the viruses, but not spores, in or on inanimate
urine but have little or no systemic antibacterial surfaces
effect: - Pasteurization: a process that kills
A. Nitrofurantoin nonsporulating microorganisms by hot water or
B. Polymyxin steam at 65-100 deg C.
C. Metronidazole
D. Fidaxomicin 5. A process that kills nonsporulating
microorganisms by hot water or steam at 65-
Urinary antiseptics: oral agent that exert antibacterial 100°C
activity in the urine but have little or no systemic A. Pasteurization
antibacterial effect: B. Antisepsis
- Nitrofurantoin C. Disinfection
- Methenamine mandelate D. Decontamination
- Methenamine hippurate
6. A cationic biguanide with very low water
2. Which urinary antiseptic may cause hemolytic solubility, not used for surgeries of the middle
anemia in patients with G6PD Deficiency? ear & neurosurgical cases:
A. Methenamine mandelate A. Alcohol
B. Methenamine hippurate B. Chlorhexidine
C. Nitrofurantoin C. Aldehyde
D. Mupirocin D. Iodine

3. This is used as a urinary antiseptic to suppress, Cationic Antimicrobials:


not treat, urinary tract infection - Quaternary ammonium compounds (QAC):
A. Methenamine mandelate benzalkonium chloride
B. Nitrofurantoin - Polymeric biguanides
C. Fosfomycin - Antibiotics such as polymyxin and Tyrocidin
D. Cotrimoxazole
Methenamine mandelate and M. hippurate used only as 7. Which of the following is commonly used as a
urinary antiseptic to suppress, not treat UTI preservative to prevent the overgrowth of
bacteria & fungi in pharmaceutical products,
laboratory sera & reagents, cosmetic products, &
contact lenses?
A. Parabens
B. Ethylene oxide
C. Hydrogen peroxide
D. Formaldehyde
Commonly used preservative agents:
- Organic acids (benzoic acid) & salts (parabens
alkyl esters of p-hydroxybenzoic acid) 11. Which of the following topical antimicrobial is
- Sorbic acid & salts used for treating impetigo and to clear chronic
- Phenolic compounds nasal carriers of staph aureus?
- Quaternary ammonium compounds, alcohols, & A. Mupirocin
mercurial (thimerosal in 0.001-0.004% conxn) B. Retapamulin
C. Fusidic acid
8. The following are major mechanisms of D. Cephalexin
antimicrobial synergism, EXCEPT:
A. Inhibition of cidal activity by static 12. Inhibits staphylococcal isoleucyl tRNA
agents synthetase:
B. Blockade of sequential steps in a A. Mupirocin
metabolic sequence B. Fusidic acid
C. Inhibition of enzymatic inactivation C. Retapamulin
D. Enhancement of antimicrobial agent D. Cephalexin
uptake
- Fusidic acid: interfering with bacterial protein
A: major mechanism of antimicrobial ANTAGONISM synthesis, specifically by preventing the
translocation of the elongation factor G (EF-G)
9. Persistent suppression of bacterial growth after from the ribosome
limited exposure to an antimicrobial agent is - Retapamulin: inhibits bacterial protein synthesis
known as the: by binding to a unique site on the active 50 S
A. Post antibiotic effect ribosomal subunits, inhibits peptidyl transferase,
B. Synergism and blocks P-site interactions
C. Antagonism - Cephalexin: inhibits bacterial cell wall synthesis
D. Concentration dependent killing
13. Recommended as the 1st line treatment for acute
- Synergism: when the inhibitory/killing of uncomplicated cystitis due to high efficacy,
two/more antimicrobials used together are minimal resistance, minimal adverse effects, low
significantly greater than expected from their propensity for collateral damage, & reasonable
effects when used individually cost:
- Antagonism: when the combined A. Nitrofurantoin
inhibitory/killing effects of two or more B. Quinolones
antimicrobial drugs are significantly less than C. Co-amoxiclav
observed when the drugs are used individually D. Cefaclor
- Concentration dependent killing: the rate and
extent of killing increase with increasing drug 14. Which urinary antiseptic is given as single dose?
concentrations A. Fosfomycin
B. Nitrofurantoin
CLINICAL CASES: C. Methenamine mandelate
10. Recommended antimicrobial in cases of D. Trimethoprim-sulfamethoxazole
extensive nonbollous impetigo:
A. Cephalexin - Nitrofurantoin: BID; long acting like macrobid or
B. Mupirocin QID (100mg)
C. Fusidic acid - Methenamine madelate: QID
D. Retapamulin - Trimethoprim: BID
B, C, D: localized for impetigo only (max of 5 lesions in
single area)
15. Which of the following should NOT be used for ANTI-CANCER
empirical treatment for acute uncomplicated 1. Chemotherapeutic agent for our patient with
cystitis due to poor efficacy & very high cervical cancer
prevalence of antimicrobial resistance? A. Cisplatin
A. Amoxicillin B. Paclitaxel
B. Nitrofurantoin C. Mitoxantrone
C. Fosfomycin D. AOTA
D. Cefuroxime 2. Secondary prevention for cervical cancer
A. Papsmear
- Nitrofurantoin: 1st line B. Adequate treatment of STI
- Fosfomycin: recommended C. HPV vaccination
- Cefuroxime: appropriate alternative when other D. Monogamous relationship
agents cannot be used (b-lactam agents
including co-amox, cefidinir, cefpodoxime, 3. Which is an example of biologic therapy
ceftibutem) A. Interferon
B. Interleukin
16. In very mild cases of C. difficile enteritis in C. Monoclonal Ab
patients who do not have other comorbidities, D. AOTA
this may be the only treatment necessary:
A. Cessation of causative antibiotics 4. The MC cancer among females in the PH is the:
B. Metronidazole as 1st line antimicrobial A. Cervical
should be started immediately B. Breast
C. Vancomycin thru IV is the recommended C. Uterine
1st line therapy D. Lung
D. Administration of probiotics
5. For certain high risk women baseline
17. Which of the following is NOT an adverse effect mammography is performed at:
of metronidazole? A. 25 y/o
A. Redneck syndrome B. 35 y/o
B. Disulfram-like reaction when taken with C. 40 y/o
alcohol D. 45 y/o
C. Neuropathy
D. Seizures 6. Chemotherapeutic agent for our patient with
breast cancer:
18. Preferred route of administration of A. Cyclophosphamide
metronidazole in the treatment of C. difficile B. Bactidaxel
enteritis: C. Cisplatin
A. Oral D. AOTA
B. Intravenous
C. Topical 7. Management plan for our breast cancer case:
D. Rectal A. Chemotherapy
B. Hormonal therapy
19. Recommended disinfectant to reduce C. Radiotherapy
transmission of Clostridium difficile infection in D. AOTA
hospitals and at home:
A. Chlorine 8. There is strong indicator of cancer if PSA result is
B. Alcohol above:
C. Phenolics A. 5 nanogram
D. Formaldehyde B. 7 nanogram
C. 9 nanogram 6. Multiple myeloma
D. 10 nanogram/mL A. FAC
B. VAD
9. This drug is used to prevent skeletal C. CHOP
complication such as pathologic fracture and D. FOLFOX
spinal cord progression in the patient with
prostate cancer: 7. Non-Hodgkins Lymphoma:
A. Bisphosphonates A. CHOP
B. Glucocorticoids B. FOLFOX
C. Mitoxantrone C. VAD
D. Cyclophosphamides D. MOPP

10. Treatment approach for the hormone refractory 8. Hodgkins Lymphoma:


state in prostatic cancer includes: A. CHOP
A. Glucocorticoids B. FOLFOX
B. Docetaxel C. VAD
C. Fluorouracil D. MOPP
D. Cyclophosphamides
9. Colorectal cancer
11. Cell cycle specific agent: A. CELOX
A. Paclitaxel B. CNF
B. Mitomycin C. VAD
C. Cisplatin D. MOPP
D. Busulfan
10. Breast cancer
12. Cell cycle non-specific agent: A. FAC
A. Busulfan B. VAD
B. Bleomycin C. CHAPS
C. Etoposide D. FOLFOX
D. Vincristine
IMMUNOPHARMACOLOGY
13. Alkylating agent used in cancer chemotherapy:
A. Methotrexate ANTIVIRAL/FUNGAL
B. Cyclophosphamide QUIZ:
C. Vincristine 2. Mechanism of action of Protease Inhibitors
D. Mitomycin A. Binds to HIV 1 & 2 integrase enzyme needed for
the integration of the virus to the host
14. Anti-metabolite agent used in cancer B. Bind directly to HIV 1 Reverse Transcriptase
chemotherapy: resulting to RNA & DNA dependent DNA
A. Methotrexate polymerase
B. Cisplatin C. Binds to host CCR5 protein which facilitates
C. Carboplatin entry of the virus inside the cell
D. Vinorelbine D. Competitive inhibition of HIV 1 reverse
transcriptase
15. Platinum analog: E. Inhibit post translational cleavage of gag-pol
A. Etoposide polyprotein resulting to immature noninfectious
B. Irinotecan virus
C. Cisplatin
D. Paclitaxel
3. Mechanism of action of NNRTI
A. Binds to CCR5 protein which facilitates entry of 9. Broadest spectrum Azole and the first Azole with
the virus inside the cell significant activity vs Mucormycosis
B. Bind directly to HIV 1 Reverse Transcriptase A. Voriconazole
resulting to RNA and DNA dependent DNA B. Itraconazole
polymerase C. Posaconazole
C. Binds to HIV 1 & 2 integrase enzyme needed for D. Isavuconazole
the integration of the virus to the host E. Fluconazole
D. Inhibit post translational cleavage of gag-pol
polyprotein resulting to immature non infectious 10. Mechanism of action is blockade of B glucan
virus synthesis
E. Competitive inhibition of HIV 1 reverse A. Flucytosine
transcriptase B. Nystatin
C. Terbinafine
4. Entry inhibitor: D. Caspofungin
A. Zidovudine NRTI E. Ketoconazole
B. Efavirenz NNRTI
C. Maraviroc 11. Clinical presentation of case 1 with
D. Ritonavir pi oropharyngeal candidiasis
E. Abacavir NRTI A. Elevated hyperemic patches on the
tongue, buccal mucosa and oropharynx
5. Used in the treatment of hepatitis B infection B. Yellowish mucopurulent discharge in the
A. Entecavir post pharyngeal wall
B. Ledipasvir C. White patches covering the buccal
C. Ombitasvir mucosa, tongue, and posterior
D. Dasabuvir pharyngeal wall
E. Daclatasvir D. Erythematous buccal mucosa and
oropharyngeal wall
6. Primarily used for CMV retinitis E. Bleeding ulcerated lesions on the tongue
A. Ganciclovir and oropharynx
B. Famcyclovir
C. Valacyclovir 12. Topical drug of choice for oropharyngeal
D. Acyclovir candidiasis
E. Pencyclovir A. Micafungin
B. Amphotericin B
7. Neuraminidase inhibitor for both Influenza A & C. Griseofulvin
B: D. Nystatin
A. Ledipasvir E. Ketoconazole
B. Ribavirin
C. Elbasvir 13. MOA of the topical drug of choice for
D. Dasabuvir oropharyngeal candidiasis
E. Peramivir A. Forms pores in cell wall membrane
B. Interfere with fungal DNA and RNA
8. Azole of choice for Aspergillus infection synthesis
A. Voriconazole C. Inhibit squalene epoxidation
B. Fluconazole D. Blocks fungal CYP450
C. Isavuconazole E. Blocks B Glucan synthase
D. Posaconazole
E. Itraconazole
14. Alternative topical drug for oropharyngeal 19. Drug of choice for otomycosis:
candidiasis A. Amphotericin B
A. Micafungin B. Griseofulvin
B. Griseofulvin C. Terbinafine
C. Caspofungin D. Clotrimazole
D. Amphotericin B E. Ketoconazole
E. Clotrimazole
20. MOA of the DOC for otomycosis (case 2):
15. Diagnosis for the case with erythematous A. Blocks B glucan synthase
maculopapular rashes on the thoracic with B. Inhibit squalene epoxidation
occasional fluid filled blisters accompanied by C. Forms pores in fungal cell wall
moderate to severe pain: D. Interfere with fungal DNA and RNA
A. CMV synthesis
B. Shingles E. Reduce ergosterol synthesis by
C. Measles inhibiting fungal CYP450
D. HIV
E. Hepatitis C PLATING:

16. True statement regarding the case with 3. Antiretroviral drugs that reduce vertical transmission
erythematous maculopapular rashes on the in HIV infection:
thoracic with occasional fluid filled blisters A. Saquinavir
accompanied by moderate to severe pain: B. Zalcitabine
A. Oral antibiotics are given as prophylaxis C. Tenofovir
to prevent secondary bacterial infection D. Efavirenz
B. Oral abacavir is the DOC
C. Antiviral creams are generally not 4. the most common adverse effect of non nucleoside
recommended transcriptase inhibitor (NNRTI) is:
D. Cytomegalovirus is the causative agent A. blood dyscrasias
E. Trfluridine creams are applied to the B. skin rashes
lesions within 12 hours to shorten C. peripheral neuropathy
healing time D. flu like syndrome

17. Used in the treatment of Herpes Zoster or 5. Adverse effect due cumulative toxicity of
Varicella Zoster Infection: amphotericin B:
A. Indinavir A. renal damage
B. Abacavir B. muscle spasm
C. Atzanavir C. vomiting
D. Tenofovir D. fever
E. Acyclovir E. hypotension

18. Causative organism in the case with edematous 6. an echinocandins that act at the fungal cell wall,
external auditory canal and fuzzy black cotton inhibiting the synthesis of 1-3-glucan, resulting in
like material: disruption of the fungal cell wall and cell death:
A. Cryptococcus A. nystatin
B. Candida B. Terbinafine
C. Aspergillus C. Caspofungin
D. Clostridium D. Griseofulvin
E. Blastomyces
7. Patient X developed psychiatric symptoms 13. Which of the following is true about
several weeks after taking antiretroviral agent. Amphotericin B
Most likely the drug is: A. Topical form not effective for mycotic
A. Etravine corneal ulcers
B. Nevirapine B. Intrathecal injection is well tolerated for
C. Efavirenz fungal meningitis
D. Delavirdine C. Completely absorbed in its oral form hence
used extensively for systemic infections
8. Which of the following is correctly paired? D. Slow intravenous form preferred for
A. Maraviroc fusion inhibitor systemic use
B. Dolutegravir integrase strand transfer E. Used as final antifungal drug after induction
inhibitor therapy using azoles for severe fungal
C. Ritonavir entry inhibitor infections
D. Enfuvirtide protease inhibitor
14. Ritonavir and lopinavir are classified as:
9. MOA of amphotericin B A. Protease inhibitors
A. Forms pores in fungal cell membrane B. Fusion inhibitors
B. Interferes with DNA and RNA synthesis in C. Integrase strand transfer inhibitors
fungi D. Entry inhibitors
C. Inhibit squalene epoxidation in fungi
D. Blocks beta glucan synthase 15. An infusion related toxicity of amphotericin B
E. Blocks fungal CYP450 and interferes with A. Azotemia
ergosterol synthesis B. Renal damage
C. Fever
10. Enzyme specifically inhibited by antriretroviral D. Arachnoiditis
agents that lead to inhibition of DNA synthesis: E. Anemia
A. HIV reverse transcriptase
B. Thymidine kinase 16. AIDS patient who received high-dosage of this
C. DNA polymerase antiviral agent chronically had an increased
D. RNA polymerase incidence of GIT disorder thrombotic
thrombocytopenic purpura (TTP) and hemolytic
11. Absorption of this drug is improved when given uremic syndrome (HUS):
with fatty foods: A. Docosanol
A. Nystatin B. Foscarnet
B. Terbinafine C. Trifluridine
C. Naftifine D. Valacyclovir
D. Griseofulvin
17. Ribavirin is primarily used for the treatment of:
12. Which of the following pairs of antiviral agent A. Influenza B infection
and clinical indication is NOT correct: B. Genital and perianal warts
A. Imiquimod for tropical treatment of genital C. Hepatitis C infection
and perianal warts D. HIV infection
B. Palivizumab for prevention of herpes
infection 18. DOC for viral keratoconjunctivitis & recurrent
C. Atazanavir for treatment of chronic epithelial keratitis:
hepatitis C infection A. Trifluridine
D. Ribavirin for the treatment of influenza B B. Valacyclovir
and A infection C. Vidarabine
D. Ganciclovir
19. The azole of choice in the treatment and
secondary prophylaxis of cryptococcal
meningitis:
A. Fluconazole
B. Ketoconazole
C. Voriconazole
D. Itraconazole

20. MOA of azoles:


A. Blocks fungal CYP450 and interferes with
ergosterol synthesis
B. Interferes with DNA and RNA synthesis in
fungi
C. Forms pores in fungal cell membrane
D. Blocks beta glucan synthase
E. Inhibit squalene epoxidation in fungi
1 It 1 =
)
0

I summation : 2 drugs i similar actin ns are taken for doubled


: combined effect of 2 drugs is greater than Effect 111-1=2 )
the sum of the effect of each
drug alone 111-1=33

1st drug counters the effect of 2nd drug by


binding to it 1 block Its action

] <2

-
short half life
-

drug goes out of the cell to produce its effect

A :
augmented
B :B 122am
C : continuous
D: delayed
E : end of use

t :
failure of therapy

drug action ID
bady
'
-

drug used in tx ,
DX I PN Of disease

-
activate receptor to maximum effect
↳ dapat urine to audio
-

antagonist Maples receptpr I PN


agonist from binding to that receptor .

overcame by t cane of agonist


1)effect when 2
drugs given together acting on
2 diff .

recep Mrs producing diff effects .


( see katzungp.tl)

↳ cimetidine is an inhibitor

] Reversible

long

-
TX for open angle glaucoma
-

tx for MG 's ileus


-

tx for MG

- DX for MG

concentration of drug that produces 50% effect


-

total concentration of receptor site


-

measured by quanta ansel graded dow


-

-
Graded nose Quanta I DDSO Effect
. LD 50 .

Therapeutic window
.

TD 50 '

El 50 / ED 50

Tl

choline ceptrr
-
-
breaks down acetylcholine


parasympathetic antagonist
parasympathinYHU


parasympathetic
SD Pag blind CK me
Siya →
sympathetic
manifestations
-
indirect acting
contraction

↳ vasidllalllr

↳ triple ] selective leukotriene Recep Mr Antagonist


response
competitive antagonist
-
µ somatic N.si

acetylcholinesterase
breaks down Ach
↳ First
Gen piperazine s

PGE i i i abortifacient
-

suppress gastric ulceration


-

PGET ; tx of Impotence
-
prom typical 51173 antagonist ; for Ntv 2° to

chemo 's radiation

You might also like